+7 495 120-13-73 | 8 800 500-97-74

(для регионов бесплатно)

Содержание

Емкость батареи конденсаторов формула

Один из наиболее важных эффектов, используемых в электронике, — ёмкость конденсаторов. Способность накапливать и хранить электрический заряд нашла применение практически во всех аналоговых цепях и логических схемах. Пассивные устройства, запасающие энергию в виде электрического поля, называли конденсаторами уже в те времена, когда учёные ещё очень мало знали о природе электричества.

История накопителей заряда

Самое раннее письменное свидетельство получения зарядов с помощью трения принадлежит учёному Фалесу из Милета (635—543 гг. до н. э.), который описал трибоэлектрический эффект от взаимодействия янтаря и сухой шерсти. Для приблизительно 2300 последующих лет любое получение электричества заключалось в трении двух различных материалов друг о друга.

Качественный рывок в знаниях о зарядах произошёл в эпоху Просвещения — период революционного развития научной мысли в образованных кругах. В это время электричество становится популярной темой, а энтузиастами было произведено немало опытов и экспериментов с генераторами на основе трения.

Первое устройство для хранения полученных зарядов было создано в 1745 г. двумя электриками (так тогда называли людей, изучающих природу статического электричества), работающими независимо друг от друга: Эвальдом фон Клейстом, деканом собора в Пруссии, и Питером ван Мюссенбруком, профессором математики и физики в университете Лейдена.

Открытие явления произошло во время опытов у обоих экспериментаторов, но с той разницей, что Мюссенбрук, во-первых, сделал немало усовершенствований первоначально созданного оборудования, а во-вторых, письменно сообщил коллегам о своих достижениях. Прошло совсем немного времени и учёные мира стали создавать накопители зарядов собственных конструкций. Это были первые шаги в эволюции конденсаторов, продолжающейся и в наши дни.

Основные даты хронологии появления устройств для хранения зарядов:

  • 1746 г. — изобретение лейденской банки в результате экспериментов по доработке устройства Клейста;
  • 1750 г. — опыты Бенджамина Франклина с батареями конденсаторов;
  • 1837 г. — публикация Майклом Фарадеем теории диэлектрической поляризации — научной основы работы накопителей;
  • конец XIX в. — начало практического применения лейденских банок вместе с первыми устройствами постоянного тока;
  • начало XX в. — изобретение слюдяных и керамических конденсаторов.

Физика ёмкостных характеристик

Устройства, обладающие способностью хранения энергии в форме электрического заряда и производящие при этом разность потенциалов, называют конденсаторами. В простейшем виде они состоят из двух или более параллельных проводящих пластин, находящихся на небольшом расстоянии друг от друга, но электрически разделённых либо воздухом, либо каким-либо другим изоляционным материалом, например, вощёной бумагой, слюдой, керамикой, пластмассой или специальным гелем.

Если подключить к пластинам источник напряжения, то одна из них получит избыток электронов, а на другой сформируется их дефицит. Ионы и электроны на каждой из этих пластин притягиваются друг к другу, но благодаря диэлектрическому барьеру они не соединяются, а накапливаются на плоскостях проводников. В результате первая пластина (электрод) окажется заряженной отрицательно, а вторая — положительно. Неподвижные заряды создают постоянное электрическое поле, теоретически сохраняемое неограниченное количество времени в незамкнутой электрической цепи.

Поток электронов на пластины называется зарядным током, продолжающим присутствовать до тех пор, пока напряжение на пластинах не сравняется с приложенным. В этот момент конденсатор считается полностью заряженным, то есть зарядов на пластинах становится так много, что они отталкивают вновь поступающие. При подключении к заряженному устройству нагрузки электроны и ионы находят новый путь друг к другу. В этом случае конденсатор работает как источник тока до момента потери разности потенциалов на электродах.

Способность конденсатора хранить заряд Q (измеряется в кулонах) называют ёмкостью.

Чем больше площадь пластин и меньше расстояние между ними (благодаря усилению эффекта притяжения зарядов между обкладками), тем большая ёмкость устройства. Степень приближения пластин ограничивается способностью диэлектрика сопротивляться разрядке пробоем между ними. Таким образом, три характеристики определяют производительность конденсатора:

  • геометрия пластин;
  • расстояние между ними;
  • диэлектрический материал между пластинами.

Единица и формулы расчёта

Ёмкость в виде электрического свойства, способного хранить заряды, измеряется в фарадах (Ф) и обозначается С. Величина названа в честь английского физика Майкла Фарадея. Конденсатор ёмкостью 1 фарад способен хранить заряд в 1 кулон на пластинах с напряжением 1 вольт. Значение С всегда положительно.

Математическое выражение фарада

Ёмкость конденсатора — постоянная величина, означающая потенциальную способность хранить энергию. Количество заряда, хранимое в отдельно взятый момент, определяется уравнением Q=CV, где V — приложенное напряжение. Таким образом, регулируя напряжение на пластинах, можно увеличивать или уменьшать заряд. Эта формула ёмкости в виде C=Q/V в единичных значениях определяет, в чём измеряется ёмкость конденсатора в СИ, и является математическим выражением фарада.

Специалисты по электронике единицу в один фарад считают не совсем практичной, поскольку она представляет собой огромное значение. Даже 1/1000 F — это очень большая ёмкость. Как правило, для реальных электрических компонентов применяют следующие величины:

  • пикофарад — 10—12 Ф;
  • нанофарад — 10—9 Ф;
  • микрофарад — 10—6 Ф.

Диэлектрическая проницаемость

Фактор, благодаря которому изолятор определяет ёмкость конденсатора, называется диэлектрической проницаемостью. Обобщённая формула расчёта ёмкости конденсатора с параллельными пластинами представлена выражением C= ε (A / d), где:

  • А — площадь меньшей пластины;
  • d — расстояние между ними;
  • ε — абсолютная проницаемость используемого диэлектрического материала.

Диэлектрическая проницаемость вакуума ε0 является константой и имеет значение 8,84х10—12 фарад на метр. Как правило, проводящие пластины разделены слоем изоляционного материала, а не вакуума. Чтобы найти ёмкость конденсатора, пластины которого находятся в воздухе, можно воспользоваться значением ε0. Разницей диэлектрической проницаемости атмосферы и вакуума можно пренебречь, поскольку их значения очень близки.

На практике в формулах нахождения ёмкости конденсатора используется относительная диэлектрическая проницаемость в качестве коэффициента, означающая, насколько электрическое поле между зарядами уменьшается в диэлектрике по сравнению с вакуумом.

Некоторые значения этой величины для различных материалов:

Поскольку эффективность конденсатора зависит от применяемого в нём изолятора, его качество как накопителя можно определить через удельную ёмкость — величину, равную отношению ёмкости к объёму диэлектрика.

Практические измерения

Значение ёмкости конденсатора обозначается на корпусе в дробных фарадах или с помощью цветового кода. Но со временем компоненты способны потерять свои качества, поэтому для некоторых критических случаев последствия могут быть неприемлемыми. Существуют и другие обстоятельства, требующие измерений. Например, необходимость знать общую ёмкость цепи или части электрооборудования. Приборов, осуществляющих непосредственное считывание ёмкости, не существует, но значение может быть вычислено вручную или интегрированными в измерительные устройства процессорами.

Для обнаружения фактической ёмкости нередко используют осциллограф как средство измерения постоянной времени (т). Эта величина обозначает время в секундах, за которое конденсатор заряжается на 63%, и равна произведению сопротивления цепи в омах на ёмкость цепи в фарадах: т=RC. Осциллограф позволяет легко определить постоянную времени и даёт возможность с помощью расчётов найти искомую ёмкость.

Существует также немало моделей любительского и профессионального электронного измерительного оборудования, оснащённого функциями для тестирования конденсаторов. Многие цифровые мультиметры обладают возможностью определять ёмкость. Эти устройства способны контролируемо заряжать и разряжать конденсатор известным током и, анализируя нарастание результирующего напряжения, выдавать довольно точный результат. Единственный недостаток большинства таких приборов — сравнительно узкий диапазон измеряемых величин.

Более сложные и специализированные инструменты — мостовые измерители, испытывающие конденсаторы в мостовой схеме. Этот метод косвенного измерения обеспечивает высокую точность. Современные устройства такого типа оснащены цифровыми дисплеями и возможностью автоматизированного использования в производственной среде, они могут быть сопряжены с компьютерами и экспортировать показания для внешнего контроля.

Идея суперконденсатора

Электричество — чрезвычайно универсальный вид энергии, обладающий одним недостатком — его трудно саккумулировать быстро. Химические батареи способны сохранять большое количество энергии, но требуют нескольких часов для полной зарядки. Этого недостатка лишены конденсаторы — они могут заряжаться практически мгновенно. Но их ёмкость не позволяет хранить большое количество энергии, поэтому весьма заманчивой выглядит идея суперконденсатора, сочетающего лучшие качества химических и электростатических накопителей электричества.

Несмотря на функциональную схожесть, аккумуляторные батареи и конденсаторы устроены совершенно по-разному. Гальванические элементы работают на принципе высвобождения электрической энергии во время химической реакции веществ внутри них. При истощении запаса активных реагентов они прекращают генерировать разность потенциалов и для нового цикла требуют инициирования током обратных химических реакций для восстановления активных веществ. Основные недостатки аккумуляторов по сравнении и конденсаторами:

  • непродолжительный жизненный цикл;
  • невысокая удельная мощность;
  • узкий диапазон температур зарядки и разрядки;
  • неспособность быстро отдать весь запас энергии.

Тем не менее обычные конденсаторы не используются в качестве активных источников напряжения из-за низкой ёмкости. Теоретические и практические суперконденсаторы (ультраконденсаторы) отличаются от обычных крайне высокой ёмкостью при большой плотности хранимой энергии, что позволяет их рассматривать как альтернативу химическим элементам.

Крупнейшие коммерческие устройства обладают ёмкостью до нескольких тысяч фарад, но их возможности всё равно несопоставимы с аккумуляторами, поэтому подобные устройства используются для хранения зарядов в течение относительно короткого периода времени. Они нашли широкое применение в качестве электрических эквивалентов механических маховиков, чтобы сглаживать напряжение источников питания, например, в ветровых турбинах или рекуперативных тормозных системах электрических транспортных средств.

Первые ультраконденсаторы появились в середине прошлого века и обладали не очень впечатляющими ёмкостями. С тех пор прогресс в совершенствовании материалов привёл к утоньшению диэлектрического слоя до одной молекулы, что позволило создавать устройства с выдающимися характеристиками. Дальнейшее развитие наноиндустрии стало основой для фундаментальных перемен в накоплении электричества. Возможно, в скором времени экологически опасные и капризные химические аккумуляторы заменят суперконденсаторы на основе молекулярно структурированных пластин и диэлектрического слоя.

1. батарея конденсаторов представляет собой два параллельно соединенных конденсаторов емкостями 3 и 6 мкФ. Определите емкость батареи

2. при условии последовательного соединения двух конденсаторов их общая емкость равна 1,2 мкФ. Емкость одного из конденсатора 2,0 мкФ. определите емкость второго конденсатора

  • Попроси больше объяснений
  • Следить
  • Отметить нарушение

Nats3 12. 09.2019

Что ты хочешь узнать?

Ответ

Объяснение:

1. батарея конденсаторов представляет собой два параллельно соединенных конденсаторов емкостями 3 и 6 мкФ. Определите емкость батареи

Сб = C1+C2 = 3 +6 = 9 мкФ

2. при условии последовательного соединения двух конденсаторов их общая емкость равна 1,2 мкФ. Емкость одного из конденсатора 2,0 мкФ. определите емкость второго конденсатора

Что такое электроемкость проводников

Если у нас есть два проводника, изолированных друг от друга, которым мы сообщаем некоторые заряды (обозначим их соответственно q 1 и q 2 ), то между ними возникнет определенная разность потенциалов. Ее величина будет зависеть от формы проводников, а также от исходных величин зарядов. Обозначим такую разность Δ φ . Если мы говорим о разности, возникающей в электрическом поле между двумя точками, то ее обычно обозначают U .

В рамках темы данной статьи нам больше всего интересна такая разность потенциалов между проводниками, когда их заряды противоположны по знаку, но равны друг другу по модулю. В таком случае мы можем ввести новое понятие – электрическая емкость (электроемкость).

Электрической емкостью системы, состоящей из двух проводников, называется отношение заряда одного проводника ( q ) к разности потенциалов между этими двумя проводниками.

В виде формулы это записывается так: C = q ∆ φ = q U .

Для измерения электрической емкости применяется единица, называемая фарад. Она обозначается буквой Ф .

Конфигурации и размеры проводников, а также свойства диэлектрика определяют величину электроемкости заданной системы. Наибольший интерес для нас представляют проводники особой формы, называемые конденсаторами.

Конденсатор – это проводник, конфигурация которого позволяет локализовать (сосредотачивать) электрическое поле в одной выделенной части пространства. Проводники, составляющие конденсатор, называются обкладками.

Если мы возьмем две плоские пластины из проводящего материала, расположим их на небольшом расстоянии друг от друга и проложим между ними слой диэлектрика, то мы получим простейший конденсатор, называемый плоским. При его работе электрическое поле будет располагаться преимущественно в промежутке между пластинами, но небольшая часть этого поля будет рассеиваться вокруг них.

Часть электрического поля вблизи конденсатора называется полем рассеяния.

Иногда в задачах мы можем не учитывать его и работать только с той частью электрического поля, которое расположено между обкладками. Однако пренебрегать полем рассеяния допустимо далеко не всегда, поскольку это может привести к ошибочным расчетам из-за нарушения потенциального характера электрического поля.

Рисунок 1 . 6 . 1 . Электрическое поле в плоском конденсаторе.

Рисунок 1 . 6 . 2 . Электрическое поле конденсатора без учета поля рассеяния, не обладающее потенциальностью.

Модуль напряженности электрического поля, которое создает каждая обкладка в плоском конденсаторе, выражается соотношением следующего вида:

Исходя из принципа суперпозиции, можно утверждать, что напряженность E → поля, которое создают обе пластины конденсатора, будет равна сумме напряженностей E + → и E — → полей каждой пластины, то есть E → = E + → + E — → .

Векторы напряженностей обеих пластин во внутренней части конденсатора будут параллельны друг другу. Значит, мы можем выразить модуль напряженности их суммарного поля в виде формулы E = 2 E 1 = σ ε 0 .

Как рассчитать электроемкость конденсатора

Вне пластин векторы напряженности будут направлены в противоположные друг от друга стороны, значит, E будет равно нулю. Если мы обозначим заряд каждой обкладки как q , а ее площадь как S , то соотношение q S даст нам представление о поверхностной плотности. Умножив E на расстояние между обкладками ( d ) , мы получим разность потенциалов между пластинами в однородном электрическом поле. Теперь возьмем оба этих соотношения и выведем из них формулу, по которой может быть рассчитана электрическая емкость конденсатора.

C = q ∆ φ = σ · S E · d = ε 0 S d .

Электрическая емкость плоского конденсатора – величина, обратно пропорциональная расстоянию между обкладками и прямо пропорциональная их площади.

Заполнение пространства между проводниками диэлектрическим материалом может увеличить электроемкость плоского конденсатора в число раз, кратное undefined.

Введем обозначение емкости в виде буквы С и запишем это в виде формулы:

Данная формула называется формулой электроемкости плоского конденсатора.

Конденсаторы бывают не только плоскими. Возможны и другие конфигурации, также обладающие специфическими свойствами.

Сферическим конденсатором называется система из 2 -х концентрических сфер, сделанных из проводящего материала, радиусы которых равны R 1 и R 2 соответственно.

Цилиндрическим конденсатором называется системы из двух проводников цилиндрической формы, длина которых равна L , а радиусы R 1 и R 2 .

Обозначим проницаемость диэлектрического материала как ε и запишем формулы, по которым можно найти электрическую емкость конденсаторов:

  • C = 4 πε 0 ε R 1 R 2 R 2 — R 1 (сферический конденсатор),
  • C = 2 π ε 0 ε L ln R 2 R 1 (цилиндрический конденсатор).

Как рассчитать электроемкость батареи конденсаторов

Если мы соединим несколько проводников между собой, то мы получим конструкцию, называемую батареей.

Способы соединения могут быть разными. Если соединение будет параллельным, то напряжение всех конденсаторов в системе будет одинаково: U 1 = U 2 = U , а заряды можно найти по формулам q 1 = С 1 U и q 2 = C 2 U . При таком соединении вся система может считаться одним конденсатором, электроемкость которого равна C , заряд – q = q 1 + q 2 , а напряжение – U . В виде формулы это выглядит так:

С = q 1 + q 2 U или C = C 1 + C 2

Если в батарее конденсаторов элементы соединены параллельно, то для нахождения общей электроемкости нам нужно сложить емкости ее отдельных элементов.

Рисунок 1 . 6 . 3 . Конденсаторы, соединенные параллельно. C = C 1 + C 2

Рисунок 1 . 6 . 4 . Конденсаторы, соединенные последовательно: 1 C = 1 C 1 + 1 C 2

Если же батарея состоит из двух последовательно соединенных конденсаторов, то заряды обоих будут одинаковы: q 1 = q 2 = q . Найти их напряжения можно так: U 1 = q C 1 и U 2 = q C 2 . Такую систему тоже можно считать одним конденсатором, заряд которого равен q , а напряжение U = U 1 + U 2 .

C = q U 1 + U 2 или 1 C = 1 C 1 + 1 C 2

Если конденсаторы в батарее соединены последовательно, то для нахождения общей электроемкости нам нужно сложить величины, обратные емкостям каждого из них.

Справедливость обеих формул, приведенных выше, не зависит от количества конденсаторов в батарее.

Рисунок 1 . 6 . 5 . Смоделированное электрическое поле плоского конденсатора.

Задачи на конденсаторы и электроемкость с решениями

Конденсатор – деталька, без которой не обойдется работа ни одного электронного прибора. Но прежде чем разбираться с основами электроники, нужно научиться решать физические задачи на конденсатор и электроемкость. Именно этим мы и займемся в сегодняшней статье, посвященной подробному разбору решений задач.

Подписывайтесь на наш телеграм: теперь помимо полезных и интересных материалов там можно найти скидки и акции на любые работы.

Задачи на конденсаторы и электроемкость с решением

Если вы не знаете, как решать задачи с конденсаторами, сначала посмотрите теорию и вспомните про памятку по решению задач по физике и полезные формулы.

Задача №1 на электроемкость батареи конденсаторов

Условие

Плоский конденсатор емкостью 16 мкФ разрезают на 4 равные части вдоль плоскостей, перпендикулярных обкладкам. Полученные конденсаторы соединяют последовательно. Чему равна емкость батaреи конденсаторов?

Решение

Из условия следует, что площадь получившихся конденсаторов в 4 раза меньше, чем у исходного. Зная это, можно найти емкость каждого полученного конденсатора:

Соединяя 4 таких конденсатора последовательно, получаем:

Ответ: 1 мкФ.

Задача №2 на энергию плоского конденсатора

Условие

Плоский конденсатор заполнили диэлектриком с диэлектрической проницаемостью, равной 2. Энергия конденсатора без диэлектрика равна 20 мкДж. Чему равна энергия конденсатора после заполнения диэлектриком? Считать, что источник питания отключен от конденсатора.

Решение

Энергия конденсатора до заполнения диэлектриком равна:

После заполнения емкость конденсатора изменится:

Энергия конденсатора после заполнения:

Ответ: 40 мкФ.

Задача №3 на последовательное и параллельное соединение конденсаторов

Условие

На рисунке изображена батарея конденсаторов. Каждый конденсатор имеет емкость 1 мкФ. Найдите емкость батареи.

Решение

Как видим, часть конденсаторов соединена параллельно, а часть последовательно. Это типичный пример смешанного соединения конденсаторов. Алгоритм решения задач при смешанном соединении конденсаторов сводится к тому, чтобы упростить схему и свести все только к параллельному или последовательному соединению. 7 м/с.

Задача №5 на вычисление энергии электрического поля конденсатора

Условие

Конденсатор подключен к источнику постоянного напряжения U=1 кВ. Емкость конденсатора равна 5 пФ. Как изменяться заряд на обкладках конденсатора и его энергия, если расстояние между обкладками уменьшить в три раза.

Решение

Заряд конденсатора равен:

Изменение заряда будет равно:

Изменение энергии:

Ответ: 5 мкДж.

Вопросы на тему «Конденсатор и электроемкость»

Вопрос 1. Что такое конденсатор?

Ответ. Конденсатор – устройство, имеющее два полюса и предназначенное для накопления электрического заряда.

Простейший тип конденсатора – плоский воздушный конденсатор. Он состоит из двух пластин (обкладок), имеющих разные заряды и разделенных воздухом. В зависимости от диэлектрика, разделяющего обкладки, разделяют:

  • воздушные конденсаторы;
  • бумажные конденсаторы;
  • слюдяные и другие конденсаторы.

Основная роль конденсатора в электронных приборах – накапливать заряд, а потом передавать его дальше в цепь.

Вопрос 2. Что такое электроемкость?

Ответ. Электроемкость – скалярная физическая величина, характеризующая способность накапливать электрический заряд. В системе СИ измеряется в Фарадах.

Вопрос 3. Какие есть способы соединения конденсаторов?

Ответ. Конденсаторы можно соединить последовательно и параллельно.

При параллельном соединении емкость цепи равна сумме емкостей отдельных конденсаторов.

При последовательном соединении величина, обратная общей емкости, равна сумме обратных емкостей каждого конденсатора.

Вопрос 4. Что такое колебательный контур?

Ответ. Это простейшая электрическая цепь, состоящая из конденсатора, катушки индуктивности и источника тока. В колебательном контуре происходят свободные электромагнитные колебания: энергия конденсатора переходит в энергию катушки, и наоборот.  

Вопрос 5. Что происходит при отключении источника питания, к которому подключен конденсатор в цепи?

Ответ. В этот момент конденсатор начинает разряжаться,  отдавая накопленный заряд другим элементам цепи.

Мы не понасылшке знаем, что от сложных задач на конденсаторы мозги буквально плавятся. Если ваш мозг устал от постоянного решения задач по физике и других заданий, обращайтесь в профессиональный образовательный сервис за консультацией и поддержкой в любое время. У нас есть решение для ваших проблем с учебой!

Калькулятор емкости последовательного соединения конденсаторов • Электротехнические и радиотехнические калькуляторы • Онлайн-конвертеры единиц измерения

Калькулятор позволяет рассчитать емкость нескольких конденсаторов, соединенных последовательно.

Пример. Рассчитать эквивалентную емкость двух соединенных последовательно конденсаторов 10 мкФ и 5 мкФ.

Входные данные

Добавить конденсатор

Выходные данные

Эквивалентная емкость

C микрофарад (мкФ)

Введите значения емкости в поля C1 и C 2, добавьте при необходимости новые поля, выберите единицы емкости (одинаковые для всех полей ввода) в фарадах (Ф), миллифарадах (мФ), микрофарадах (мкФ), пикофарадах (пФ), нанофарадах (нФ) и нажмите на кнопку Рассчитать.

1 мФ = 0,001 Ф. 1 мкФ = 0,000001 = 10⁻⁶ Ф. 1 нФ = 0,000000001 = 10⁻⁹ Ф. 1 пФ = 0,000000000001 = 10⁻¹² Ф.

В соответствии со вторым правилом Кирхгофа, падения напряжения V₁, V₂ and V₃ на каждом из конденсаторов в группе из трех соединенных последовательно конденсаторов в общем случае различные и общая разность потенциалов V равна их сумме:

По определению емкости и с учетом того, что заряд Q группы последовательно соединенных конденсаторов является общим для всех конденсаторов, эквивалентная емкость Ceq всех трех конденсаторов, соединенных последовательно, определяется как

или

Для группы из n соединенных последовательно конденсаторов эквивалентная емкость Ceq равна величине, обратной сумме величин, обратных емкостям отдельных конденсаторов:

или

Эта формула для Ceq и используется для расчетов в этом калькуляторе. Например, общая емкость соединенных последовательно трех конденсаторов емкостью 10, 15 and 20 мкФ будет равна 4,62 мкФ:

Если конденсаторов только два, то их общая емкость определяется по формуле

или

Если имеется n соединенных последовательно конденсаторов с емкостью C, их эквивалентная емкость равна

Отметим, что для расчета общей емкости нескольких соединенных последовательно конденсаторов используется та же формула, что и для расчета общего сопротивления параллельно соединенных резисторов.

Отметим также, что общая емкость группы из любого количества последовательно соединенных конденсаторов всегда будет меньше, чем емкость самого маленького конденсатора, а добавление конденсаторов в группу всегда приводит к уменьшению емкости.

Конденсаторы на печатной плате

Отдельного упоминания заслуживает падение напряжения на каждом конденсаторе в группе последовательно соединенных конденсаторов. Если все конденсаторы в группе имеют одинаковую номинальную емкость, падение напряжения на них скорее всего будет разным, так как конденсаторы в реальности будут иметь разную емкость и разный ток утечки. На конденсаторе с наименьшей емкостью будет наибольшее падение напряжения и, таким образом, он будет самым слабым звеном этой цепи.

Выравнивающие резисторы уменьшают разброс напряжений на отдельных конденсаторах

Для получения более равномерного распределения напряжений параллельно конденсаторам включают выравнивающие резисторы. Эти резисторы работают как делители напряжения, уменьшающие разброс напряжений на отдельных конденсаторах. Но даже с этими резисторами все равно для последовательного включения следует выбирать конденсаторы с большим запасом по рабочему напряжению.

Если несколько конденсаторов соединены параллельно, разность потенциалов V на группе конденсаторов равна разности потенциалов соединительных проводов группы. Общий заряд Q разделяется между конденсаторами и если их емкости различны, то заряды на отдельных конденсаторах Q₁, Q₂ and Q₃ тоже будут различными. Общий заряд определяется как

Конденсаторы, соединенные параллельно

По определению емкости, эквивалентная емкость группы конденсаторов равна

отсюда

или

Для группы n включенных параллельно конденсаторов

То есть, если несколько конденсаторов включены параллельно, их эквивалентная емкость определяется путем сложения емкостей всех конденсаторов в группе.

Возможно, вы заметили, что конденсаторы ведут себя противоположно резисторам: если резисторы соединены последовательно, их общее сопротивление всегда будет выше сопротивлений отдельных резисторов, а в случае конденсаторов всё происходит с точностью до наоборот.

Конденсаторы на печатной плате

Глава 20. Конденсаторы

Для накопления разноименных электрических зарядов служит устройство, которое называется конденсатором. Конденсатор — система двух изолированных друг от друга проводников (которые часто называют обкладками конденсатора), один из которых заряжен положительным, второй — таким же по величине, но отрицательным зарядом. Если эти проводники представляют собой плоские параллельные пластинки, расположенные на небольшом рас-стоянии друг от друга, то конденсатор называется плоским.

Для характеристики способности конденсатора накапливать заряд вводится понятие электроемкости (часто говорят просто емкости). Емкостью конденсатора называется отношение заряда конденсатора к той разности потенциалов , которая возникает между обкладками при их заряжении зарядами и (эту разность потенциалов проводников часто называют электрическим напряжением между обкладками и обозначают буквой ):

(20.1)

Поскольку величины и (или ) в формуле (20.1) зависимы, то емкость (20.1) не зависит от и , а является характеристикой геометрии системы проводников. Действительно, при сообщении проводникам зарядов и проводники приобретут потенциалы, разность которых будет пропорциональна заряду . Поэтому в отношении (20.1) заряд сокращается.

Выведем формулу для емкости плоского конденсатора (эта формула входит в программу школьного курса физики). При заряжении параллельных пластин, расположенных на небольшом расстоянии друг от друга, зарядами и , в пространстве между ними возникает однородное электрическое поле с напряженностью (см. гл. 18):

(20.2)

Разность потенциалов между пластинами равна

(20.3)

где — площадь пластин, — расстояние между ними. Отсюда, вычисляя отношение заряда к разности потенциалов (20.3), находим емкость плоского конденсатора

(20.4)

Если все пространство между обкладками заполнено диэлектриком с диэлектрической проницаемостью , то поле (20.2) и разность потенциалов (20.3) убывает в раз, а емкость конденсатора в раз взрастает

(20.5)

Для конденсаторов, соединенных в батареи, вводится понятие эквивалентной емкости, как емкости одного конденсатора, который при заряжении его тем же зарядом, что и батарея дает ту же разность потенциалов, что и батарея конденсаторов. Приведем формулы для эквивалентной емкости, а также для заряда и электрического напряжения на каждом конденсаторе при последовательном и параллельном их соединении.

Последовательное соединение (см. рисунок). При сообщении левой пластине левого конденсатора заряда , а правой пластине правого заряда , на внутренних пластинах благодаря поляризации будут индуцироваться заряды (см. рисунок; значения индуцированных зарядов приведены под пластинами). Можно доказать, что в результате поляризации каждый конденсатор будет заряжен такими же зарядами и , как и заряды крайних пластин, напряжение на всей батарее конденсаторов равно сумме напряжений на каждом, а обратная эквивалентная емкость батареи — сумме обратных емкостей всех конденсаторов

(20.6)

Параллельное соединение (см. рисунок). В этом случае если сообщить левому проводнику заряд , правому сообщить заряд , заряд распределится между конденсаторами, вообще говоря, не одинаково, но по закону сохранения заряда .

Поскольку правые пластины всех конденсаторов соединены между собой, левые — тоже, то они представляют собой единые проводники, и, следовательно, разность потенциалов между пластинами каждого конденсатора будет одинакова: . Можно доказать, что при таком соединении конденсаторов эквивалентная емкость батареи равна сумме емкостей отдельных конденсаторов

(20.7)

Заряженный конденсатор обладает определенной энергией. Если конденсатор емкости заряжен зарядом , то энергия этого конденсатора (можно говорить энергия электрического поля конденсатора) равна

(20.8)

С помощью определения электрической емкости (20.1) можно переписать формулу (20.8) еще в двух формах:

(20.9)

Рассмотрим в рамках этого минимума сведений о конденсаторах типичные задачи ЕГЭ по физике, которые были предложены в первой части книги.

Электроемкость конденсатора — его геометрическая характеристика, которая при неизменной геометрии не зависит от заряда конденсатора (задача 20.1.1 — ответ 3). Аналогично не меняется емкость конденсатора при увеличении напряжения на конденсаторе (задача 20.1.2 — ответ 3).

Связь между единицами измерений (задача 20.1.3) следует из определения емкости (20.1). Единица электрической емкости в международной системе единиц измерений СИ называется Фарада. 1 Фарада — это емкость такого конденсатора, между пластинами которого возникает напряжение 1 В при зарядах пластин 1 Кл и -1 Кл (ответ 4).

Поскольку электрическое поле в плоском конденсаторе однородно, то напряженность поля в конденсаторе и напряжение между пластинами связаны соотношением (см. формулу (18.9)) , где — расстояние между пластинами. Отсюда находим напряженность поля между обкладками плоского конденсатора в задаче 20.1.4

(ответ 4).

Согласно определению электрической емкости имеем в задаче 20.1.5

(ответ 2).

Из формулы (20.4) для емкости плоского конденсатора заключаем, что при увеличении площади его пластин в 3 раза (задача 20.1.6) его емкость увеличивается в 3 раза (ответ 1).

При уменьшении в раз расстояния между пластинами емкость плоского конденсатора возрастет в раз. Поэтому новое напряжение на конденсаторе (задача 20.1.7) можно найти из следующей цепочки формул

где и — новый заряд конденсатора (ответ 3).

Так как конденсатор в задаче 20.1.8 подключен к источнику, то между его пластинами поддерживается постоянное напряжение независимо от расстояния между ними. Поэтому заряд конденсатора изменяется при раздвигании пластин так же, как изменяется его емкость. А поскольку при увеличении расстояния между пластинами вдвое емкость конденсатора уменьшается вдвое (см. формулу (20.4)), то вдвое уменьшается и заряд конденсатора (ответ 2).

В задаче 20.1.9 конденсатор отключен от источника в процессе сближения пластин. Поэтому не меняется их заряд. А поскольку напряженность электрического поля между пластинами определяется соотношением (20.2)

то напряженность электрического поля между пластинами также не изменяется (ответ 3). Этот же результат можно получить и через определение емкости с учетом того, что

произведение от расстояния между пластинами не зависит (см. формулу (20.4)).

Из формул (20.8), (20.9) видим, что только одно из приведенных в качестве ответов к задаче 20.1.10 соотношений (а именно — 2) определяет энергию конденсатора.

При последовательном соединении конденсаторов (задача 20.2.1) одинаковыми будут их заряды независимо от значений их электрических емкостей (ответ 2). При параллельном соединении конденсаторов (задача 20.2.2) одинаковыми будут напряжения на каждом из них (ответ 3).

Поскольку конденсатор в задаче 20.2.3 отключен от источ-ника напряжения, его заряд не меняется в процессе раздвигания пластин. Поэтому для исследования изменения энергии конденсатора удобно воспользоваться формулой (20.8)

(1)

Так как при увеличении расстояния между пластинами в раз электрическая емкость конденсатора уменьшается в раз, то согласно формуле (1) энергия конденсатора увеличится в раз (ответ 1).

В задаче 20.2.4 не изменяется напряжение на конденсаторе. Поэтому воспользуемся первой из формул (20.9)

Из этой формулы заключаем, что при увеличении в раз расстояния между пластинами энергия конденсатора уменьшится в раз — ответ 2. (Разница с предыдущей задачей связана с тем, что здесь кроме внешних сил, совершающих работу при раздвигании пластин, совершает работу источник напряжения.)

В задаче 20.2.5 изменяют расстояние между пластинами (и, следовательно, емкость) и заряд конденсатора. Поэтому удобно воспользоваться формулой (20.8)

Из этой формулы заключаем, что при увеличении расстояния между пластинами в 2 раза и увеличении заряда конденсатора в 2 раза его энергия возрастет в 8 раз (ответ 4).

Поскольку в задаче 20.2.6 конденсаторы соединены последовательно, емкость батареи конденсаторов можно найти по формуле (20.6), откуда находим емкость батареи конденсаторов (ответ 2).

В задаче 20.2.7 конденсаторы соединены параллельно, поэтому емкость батареи конденсаторов можно найти по формуле (20.7): (ответ 2).

Основной вопрос, на который нужно ответить в задаче 20.2.8, это как соединены конденсаторы? Последовательно, параллельно, по-другому? Попробуем по-другому расположить в пространстве и изменить длину соединительных проводов, чтобы схема стала более понятной. Очевидно, что можно соединить вершину 1 и вершину 3 («уменьшив» длину провода 1-3), а также вершины 2 и 4. При этом средний конденсатор разворачивается в пространстве, и схема приобретает вид, показанный на рисунке, откуда видно, что конденсаторы соединены параллельно. Поэтому (ответ 1).

Когда в заряженный плоский конденсатор вставляют металлическую пластинку (задача 20.2.9), параллельную обкладкам конденсатора, напряженность электрического поля внутри пластинки становится равным нулю, вне пластинки между обкладками конденсатора остается таким же, каким оно было в отсутствие пластинки , где — заряд конденсатора, — площадь его пластин. Поэтому напряжение между обкладками конденсатора определяется соотношением:

где — расстояние между обкладками конденсатора, — толщина пластинки. Отсюда находим емкость рассматриваемого конденсатора

(ответ 4).

Чтобы найти емкость сферического конденсатора (задача 20.2.10) сообщим его обкладкам заряды и , найдем напряжение между обкладками, вычислим отношение заряда к напряжению. Разность потенциалов двух концентрических сфер, заряженных зарядами и (напряжение между обкладками сферического конденсатора), определена в задаче 19.2.5., откуда находим электрическую емкость сферического конденсатора (ответ 3):

Методика расчёта электрических цепей с конденсаторами.

Методика расчёта электрических цепей с конденсаторами.

Краткие теоретические сведения:

Электроемкостью системы из двух проводников называется физическая величина, определяемая как отношение заряда q одного из проводников к разности потенциалов Δφ между ними:

В системе СИ единица электроемкости называется фарад (Ф): 

Конденсаторы могут соединяться между собой, образуя батареи конденсаторов. При параллельном соединении конденсаторов (рис. 1.) напряжения на конденсаторах одинаковы: U1 = U2 = U, а заряды равны q1 = С1U и q2 = C2U. Такую систему можно рассматривать как единый конденсатор электроемкости C, заряженный зарядом q = q1 + q2при напряжении между обкладками равном U. Отсюда следует 

Таким образом, при параллельном соединении электроемкости складываются.

Рисунок 1.

Параллельное соединение конденсаторов. C = C1 + C2

Рисунок 2.

Последовательное соединение конденсаторов. 

При последовательном соединении (рис.2.) одинаковыми оказываются заряды обоих конденсаторов: q1 = q2 = q, а напряжения на них       Такую систему можно рассматривать как единый конденсатор, заряженный зарядом q при напряжении между обкладками U = U1 + U2. Следовательно, 

При последовательном соединении конденсаторов складываются обратные величины емкостей.

Пример расчёта:На рисунке 2 приведена схема соединения конденсаторов. Определить эквивалентнуюемкостьСэкв батареи конденсаторов, общий заряд Q, напряжение сети U, напряжение и заряд на каждом конденсаторе, если дано: C1=24 мкФ; С2=С3=8 мкФ; С4=12 мкФ; С5=6 мкФ; напряжение на пятом конденсаторе U5=30 В.

Рисунок 3

Дано: C1=24 мкФ; С2=С3=8 мкФ; С4=12 мкФ; С5=6 мкФ; U5=30 ВОпределить: U, Q, Сэкв, U1, U2, U3, U4, Q1.

Решение: 1. Общая емкость последовательно соединенных конденсаторов С4 и С5:

2. Общая емкость параллельно соединенных конденсаторов С3 иС4,5:

3. Общая емкость последовательно соединенных конденсаторов С1, С2 и С3,4,5, которая и является эквивалентной емкостью батареи конденсаторов:

4. По заданному напряжению U5 и емкости конденсатора С5 определяем заряд, накапливаемый этим конденсатором:

5. Заряд конденсатора С4 Q4=Q5=Q4,5=180・10-6 Кл, т. к. конденсаторы С4 и С5 соединены последовательно. 6. Напряжение на четвертом конденсаторе:

7. Напряжение на третьем конденсаторе:

8. Заряд конденсатора С3:

9. Общий заряд батареи и заряды конденсаторов С1 и С2:

10. Напряжение на первом и втором конденсаторах:

11. Напряжение сети (напряжение последовательно соединенных конденсаторов С1, С2, С3,4,5):

12. Энергия электрического поля батареи:

Выполнить задание:

На рисунке 4 дана схема соединения конденсаторов. Значение емкостей конденсаторов и значение одного из напряжений или зарядов для своего варианта взять из таблицы 1.Вычислить эквивалентную емкость батареи конденсаторов; напряжение сети, напряжение на каждом конденсаторе; общий заряд и заряд на каждом конденсаторе; энергию, накопленную батареей, а также потенциал заданной точки.

Рисунок 4.

Таблица 1

№ вар.

Емкость конденсатора, мкФ

Напряжение,

заряд

Точка, потенциал которой следует вычислить

С1

С2

С3

С4

С5

1

120

280

16

80

70

U=20 В

Б

2

600

200

150

400

200

Q3=72∙10-4 Кл

Б

3

24

12

2

16

14

U5=25 В

А

4

30

20

12

20

16

Q4=4∙10-4 Кл

Б

5

10

15

24

6

9

U1=15 В

А

6

12

6

5

9

9

Q2=282∙10-6Кл

А

7

30

15

10

65

15

Q5=6∙10-4 Кл

А

8

18

9

12

15

21

U2=84 В

Б

9

140

60

6

30

18

U3=50 В

А

10

150

50

37,5

30

20

Q1=3∙10-4 Кл

Б

Ответить на контрольные вопросы:

1.От чего зависит ёмкость конденсатора ?

2.Как изменится ёмкость батареи конденсаторов, если вместо последовательного соединения их соединили параллельно?

3.Изменится ли ёмкость воздушного конденсатора, если раздвинуть пластины так, чтобы расстояние между ними увеличилось с 5 до 14 мм?

Задачи по физике и математике с решениями и ответами

Задача по физике — 4029

Из проволоки сделан куб, в каждое ребро которого вставлен конденсатор с емкостью $C$. Куб подключен к цепи противоположными вершинами, как показано на рисунке. Определите емкость $C_{0}$ получившейся батареи конденсаторов.
Подробнее

Задача по физике — 4030

Два одинаковых плоских воздушных конденсатора соединены последовательно, и к ним подведено постоянное напряжение $U$. Первый конденсатор заполняют диэлектриком с диэлектрической проницаемостью $\epsilon$. Во сколько раз изменится напряженность электрического поля в каждом из конденсаторов? Подробнее

Задача по физике — 4031

Два конденсатора, имеющие емкости $C_{1} = 10 мкФ$ и $C_{2} = 50 мкФ$, соединены последовательно. Каждый из них способен выдержать напряжение соответственно $U_{1}$ и $U_{2}$. Какое наибольшее напряжение $U_{max}$ может выдержать батарея из этих конденсаторов? Решите задачу при: a) $U_{1} = U_{2} = 120 В$; б) $U_{1} = 120 В, U_{2} = 6 В$. Подробнее

Задача по физике — 4033

В схеме, изображенной на рисунке, емкость каждого конденсатора равна $C$. Вначале ключ разомкнут, конденсатор 1 заряжен до напряжения $U_{0}$, остальные конденсаторы не заряжены. Определите напряжение на каждом из конденсаторов после замыкания ключа.
Подробнее

Задача по физике — 4034

Две одинаковые металлические квадратные пластины размерами $a \times a$ находятся на расстоянии $d \ll a$ друг от друга. Одна из пластин имеет заряд $+ 3Q$, а другая — заряд $+Q$. Определите напряжение $U$ между пластинами. Как разместятся заряды на каждой из пластин? Подробнее

Задача по физике — 4035

Найдите разность потенциалов $U$ на выходе цепи (см. рисунок), если на вход подано напряжение $U_{0} = 80 В$. Емкости конденсаторов: $C_{1} = 1 мкФ, C_{2} = 2 мкФ$.
Подробнее

Задача по физике — 4036

В плоский конденсатор помещают две параллельные тонкие металлические пластины на одинаковом расстоянии друг от друга и от обкладок конденсатора (см. рисунок). На обкладки конденсатора подано напряжение $U$, обкладка 1 заземлена. 1) Каковы потенциалы пластин 2 и 3? 2) Как изменятся потенциалы пластин и напряженность поля во всех трех промежутках, если пластины 2 и 3 на короткое время замкнуть проволокой? 3) Во сколько раз изменяется емкость конденсатора при замыкании пластин 2 и 3? Изменяются ли при этом заряды на обкладках 1 и 4?
Подробнее

Задача по физике — 4037

Конденсатор подключен к аккумулятору. Как изменится энергия конденсатора при раздвигании его пластин? Как согласуется это изменение с законом сохранения энергии? Каким будет ответ в случае, если заряженный конденсатор отключен от аккумулятора перед раздвиганием пластин? Подробнее

Задача по физике — 4038

Какое количество теплоты $Q$ выделится в цепи при переводе ключа из положения 1 в положение 2 (см. рисунок)? Энергией электромагнитного излучения можно пренебречь.
Подробнее

Задача по физике — 4039

Капля ртути, заряженная до потенциала $\phi_{0}$, распадается на $N$ одинаковых капель с одинаковыми зарядами. Капли разлетаются на большое расстояние друг от друга. Определите потенциал $\phi$ каждой из образовавшихся капель. Подробнее

Задача по физике — 4040

Четыре одинаковых шарика с одинаковыми одноименными зарядами $q$ (см. рисунок) связаны одинаковыми нерастяжимыми нитями. Докажите, что равновесие достигается, когда шарики располагаются в вершинах квадрата. На шарики действуют только кулоновские силы и силы натяжения нитей.
Подробнее

Задача по физике — 4041

Внутри гладкой диэлектрической сферы радиуса $R$ находится маленький шарик массы $m$ с зарядом $+q$.{ \circ}$ к пластинам. Энергия электрона $W = 1500 эВ$, расстояние между пластинами $d = 10 мм$. При каком напряжении $U_{1}$ на конденсаторе электрон вылетит параллельно пластинам конденсатора? Каким будет ответ, если длину конденсатора увеличить до $l_{2} = 10 см$? Подробнее

Электрическая емкость. Конденсаторы

⇐ ПредыдущаяСтр 5 из 10Следующая ⇒

 

Т5.138 Если заряд каждой из обкладок конденсатора увеличить в n раз, то его электроемкость

1) увеличится в n раз 2) уменьшится в n раз

3) не изменится 4) увеличится в n2 раз

Т5.139 (а) Пространство между обкладками плоского воздушного конденсатора заполнили диэлектриком с диэлектрической проницаемостью e = 3. Как изменилась емкость конденсатора?

1) уменьшилась в 3 раза 2) увеличилась в 3 раза

3) уменьшилась в 6 раз 4) увеличилась в 6 раз

 

(б) Расстояние между обкладками плоского воздушного конденсатора увеличилось в 2 раза. Как изменилась емкость конденсатора?

1) уменьшилась в 2 раза 2) увеличилась в 2 раза

3) уменьшилась в 4 раза 4) увеличилась в 4 раза 5) не изменилась

Т5.140 Конденсатор переменной емкости подключен к источнику напряжения. Как нужно изменить емкость конденсатора, чтобы заряд на его обкладках увеличился в 4 раза?

1) уменьшить в 4 раза 2) уменьшить в 2 раза 3) увеличить в 4 раза

4) увеличить в 2 раза 5) уменьшить в 8 раз

Т5.141 Конденсатор переменной емкости зарядили и отключили от источника. Как нужно изменить емкость конденсатора, чтобы разность потенциалов между его обкладками увеличилась в 4 раза?

1) уменьшить в 4 раза 2) уменьшить в 2 раза 3) увеличить в 4 раза

4) увеличить в 2 раза 5) уменьшить в 8 раз

Т5.142 Плоский конденсатор зарядили до разности потенциалов 80 В и отключили от источника. Какой будет разность потенциалов между пластинами, если расстояние между ними уменьшить в 2 раза?

1) 200 В 2) 150 В 3) 160 В 4) 210 В 5) 40 В

Т5.143 Плоский конденсатор зарядили до разности потенциалов 220 В и отключили от источника. На сколько изменится разность потенциалов между пластинами, если расстояние между ними увеличить в 2 раза?

1) 20 В 2) 60 В 3) 220 В 4) 440 В 5) 110 В

Т5.144 Разность потенциалов заряженного и отсоединенного от источника плоского конденсатора удвоилась, когда заполнявший его диэлектрик удалили. Определите диэлектрическую проницаемость диэлектрика.

1) 2 2) 2,5 3) 4 4) 6 5) 8

Т5.145 Разность потенциалов между пластинами плоского конденсатора равно 720 В, а расстояние между ними 6 мм. Определите напряженность поля внутри конденсатора.

1) 120 В/м 2) 1,2·106 В/м 3) 1,2·105 В/м 4) 4320 В/м

Т5.146 Определите расстояние между обкладками плоского конденсатора, если к ним приложена разность потенциалов 3000 В, а напряженность поля внутри конденсатора равна 7,5·104 В/м.

1) 4·10-2 м 2) 0,025 м 3) 0,25 м 4) 2,25·10-2 м 5) 5,5·10-3 м

Т5.147 Найдите расстояние между обкладками плоского конденсатора емкостью 4·10-5 Ф, если заряд конденсатора 4·10-4 Кл, а напряженность электрического поля внутри него равна 5000 В/м.

1) 0,003 м 2) 0,004 м 3) 0,002 м 4) 0,005 м 5) 0,001 м

Т5.148 Напряженность поля между обкладками плоского воздушного конденсатора, подключенного к источнику напряжения, равна 100 В/м. Какой станет величина напряженности, если расстояние между обкладками увеличить в 4 раза?

1) 400 В/м 2) 200 В/м 3) 25 В/м 4) 30 В/м 5) 150 В/м

Т5.149 Между вертикально расположенными обкладками плоского конденсатора поместили подвешенный на нити шарик массой 2·10-3 кг и зарядом 10-6 Кл. Определите тангенс угла отклонения нити от вертикали при подаче на конденсатор напряжения 400 В. Расстояние между обкладками 0,01 м.

1) 0,5 2) 1 3) 1,5 4) 2 5) 3

Т5.150 Емкость одного конденсатора в 5 раз больше емкости второго. Найдите отношение емкости параллельного соединения этих конденсаторов к емкости их последовательного соединения.

1) 4,2 2) 5,2 3) 6,2 4) 7,2 5) 8,2

Т5.151 Найдите емкость конденсатора, изображенного на рисунке. Площадь пластин 4,7·10-4 м2, d1 = 2 см, d2 = 1 см, ε1 = 4, ε2 = 3.

1) 2,5·10-13 Ф 2) 5·10-13 Ф 3) 6·10-13 Ф

4) 8·10-13 Ф 5) 3·10-13 Ф

 

Т5.152 Конденсатор емкостью С расположен вертикально. Нижнюю половину пространства между обкладками конденсатора заполнили диэлектриком с проницаемостью ε = 4. Чему стала равной емкость конденсатора?

1) 4С 2) 2С 3) 2,5С 4) 5С 5) 3С

Т5.153 Конденсатор емкостью С расположен горизонтально. Нижнюю половину пространства между обкладками конденсатора заполнили диэлектриком с проницаемостью ε = 3. Чему стала равной емкость конденсатора?

1) 1,5С 2) 2С 3) 2,5 С 4) 3С 5) 3,5С

Т5.154 Плоский конденсатор емкостью 16 мкФ разрезали вдоль плоскостей, перпендикулярных обкладкам, на 4 одинаковые части, и полученные конденсаторы соединили последовательно. Чему равна емкость такой батареи конденсаторов?

1) 16 мкФ 2) 4 мкФ 3) 1 мкФ 4) 64 мкФ 5) 256 мкФ

Т5.155 Четыре одинаковых конденсатора емкостью 15 нФ каждый спаяны в виде квадратного каркаса и включены во внешнюю цепь противоположными вершинами. Чему равна емкость такого соединения?

1) 7,5 нФ 2) 15 нФ 3) 30 нФ 4) 60 нФ 5) 3,75 нФ

Т5.156 Четыре одинаковых конденсатора емкостью 15 нФ каждый спаяны в виде квадратного каркаса и включены во внешнюю цепь соседними вершинами. Чему равна емкость такого соединения?

1) 20 нФ 2) 12,5 нФ 3) 15 нФ 4) 60 нФ 5) 3,74 нФ

Т5.157 Емкость участка цепи, состоящего из трех одинаковых конденсаторов соединенных последовательно, равна 2 мкФ. Найдите емкость между точками АВ участка цепи, составленного из тех же конденсаторов, если их соединили так, как показано на рисунке.

1) 6 мкФ 2) 4 мкФ 3) 9 мкФ 4) 2 мкФ

Т5.158 Два конденсатора с емкостями С1 = 3 мкФ и С2 = 6 мкФ соединены последовательно и подключены к источнику постоянного напряжения 9 В. Чему равен заряд первого конденсатора?

1) 6 мкКл 2) 9 мкКл 3) 12 мкКл 4) 15 мкКл 5) 18 мкКл

Т5.159 Два конденсатора с емкостями С1 = 4 мкФ и С2 = 1 мкФ соединены последовательно и подключены к источнику постоянного напряжения 220 В. Как распределится напряжение между конденсаторами?

1) U1 = 44 B, U2 = 176 B 2) U1 = 176 B, U2 = 44 B

3) U1 = 55 B, U2 = 165 B 4) U1 =165 B, U2 = 55 B

Т5.160 Определите величину заряда, который нужно сообщить трем параллельно соединенным конденсаторам, чтобы зарядить их до разности потенциалов 20 В, если емкости конденсаторов 0,5 мкФ, 1 мкФ и 1,5 мкФ.

1) 20 мкКл 2) 30 мкКл 3) 40 мкКл 4) 50 мкКл 5) 60 мкКл

Т5.161 Латунные листы с проложенными между ними стеклянными пластинами образуют батарею конденсаторов. Определите емкость этой батареи, если число латунных листов равно 5, площадь каждого листа равна площади стеклянной пластины и составляет 200 см2, толщина каждой пластины 2 мм, а диэлектрическая проницаемость стекла равна 7.

1) 30,9·10-10 Ф 2) 24,8·10-10 Ф 3) 1,55·10-10 Ф 4) 1,24·10-10 Ф

Т5.162 Чему равна электроемкость показанного на рисунке соединения конденсаторов?

1) 4С 2) 2С 3) С 4) С/2 5) 2С/5

 

Т5.163 Чему равна емкость батареи конденсаторов, изображенных на рисунке?

1) 5С 2) 4С 3) 3С 4) 1,5С 5) 2С

 

 

Т5.164 Чему равна емкость батареи конденсаторов, изображенных на рисунке? С = 10 нФ.

1) 100 нФ 2) 84 нФ 3) 62 нФ 4) 4,8 нФ

 

Т5.165 Найдите емкость батареи конденсаторов (см. рисунок), если С = 5 нФ.

1) 5 нФ 2) 10 нФ 3) 15 нФ 4) 20 нФ 5) 25 нФ

 

 

Т5.166 Общий заряд батареи конденсаторов q = 20 мкКл. Найдите заряд на конденсаторе емкостью 2С.

1) 8 мкКл 2) 2 мкКл 3) 4 мкКл

4) 6 мкКл 5) 1 мкКл

Т5.167 К точкам А и В батареи конденсаторов подключают источник с напряжением на клеммах U = 50 В. Найдите величину заряда на конденсаторе емкостью 3С. С = 0,1 мкФ.

1) 2 мкКл 2) 4 мкКл 3) 6 мкКл 4) 8 мкКл 5) 10 мкКл

Т5.168 В цепи, изображенной на рисунке, найдите заряд на конденсаторе С2. Емкости С1 = С2 = С3 = 3·10-6 Ф. Напряжение на батарее 100 В.

1) 0,0001 Кл 2) 0,0002 Кл

3) 0,0003 Кл 4) 0,0004 Кл 5) 0,0005 Кл

Т5.169 Конденсатор емкостью С1 = 7,7 мкФ зарядили до разности потенциалов 125 В и отключили от источника питания. После того, как к нему параллельно присоедини незаряженный конденсатор емкостью С2, напряжение на конденсаторе С1 упало до 25 В. Чему равна емкость С2?

1) 15,4 мкФ 2) 23,1 мкФ 3) 25,3 мкФ 4) 30,8 мкФ 5) 38,5 мкФ

Т5.170 Конденсатор, заряженный до разности потенциалов 40 В, соединяют одноименно заряженными обкладками с конденсатором удвоенной емкости. Чему была равна разность потенциалов между обкладками второго конденсатора до соединения, если установившаяся разность потенциалов на батарее конденсаторов стала равной 20 В.

1) 10 В 2) 20 В 3) 40 В 4) 60 В 5) 100 В

Т5.171 Энергия заряженного конденсатора емкостью 0,02 мФ равна 0,001 Дж. Найдите напряжение между обкладками конденсатора.

1) 10 В 2) 12 В 3) 15 В 4) 20 В 5) 25 В

Т5.172 Найдите емкость конденсатора, если для того, чтобы зарядить его до напряжения 10 В, необходимо совершить работу 0,001 Дж.

1) 2·10-6 Ф 2) 2·10-5 Ф 3) 4·10-5 Ф 4) 5·10-6 Ф 5) 4·10-6 Ф

Т5.173 До какого напряжения заряжен конденсатор, если его заряд равен 0,2 мКл, а энергия конденсатора равна 0,001 Дж?

1) 40 В 2) 20 В 3) 15 В 4) 10 В 5) 5 В

Т5.174 Во сколько раз увеличится энергия заряженного конденсатора при увеличении приложенного к нему напряжения на 10%?

1) 1,21 2) 1,44 3) 1,69 4) 1,96 5) 2,25

Т5.175 Конденсатор переменной емкости зарядили и отключили от источника. Как нужно изменить его емкость, чтобы энергия поля в конденсаторе увеличилась в 4 раза?

1) уменьшить в 8 раз 2) уменьшить в 2 раза

3) увеличить в 4 раза 4) увеличить в 2 раза 5) уменьшить в 4 раза

Т5.176 Конденсатор переменной емкости подключен к источнику напряжения. Как нужно изменить его емкость, чтобы энергия поля в конденсаторе увеличилась в 4 раза?

1) уменьшить в 4 раза 2) уменьшить в 2 раза

3) увеличить в 4 раза 4) увеличить в 2 раза 5) уменьшить в 8 раз

Т5.177 Как изменится энергия подключенного к источнику плоского конденсатора, если расстояние между его пластинами увеличить в 2 раза?

1) увеличится в 2 раза 2) увеличится в 4 раза

3) не изменится 4) уменьшится в 2 раза 5) уменьшится в 4 раза

Т5.178 Как изменится энергия плоского воздушного конденсатора, заряжен-ного и отсоединенного от батареи, если пространство между его обклад-ками заполнить диэлектриком (ε = 2)?

1) увеличится в 2 раза 2) увеличится в 4 раза

3) не изменится 4) уменьшится в 2 раза 5) уменьшится в 4 раза

Т5.179 Два одинаковых воздушных конденсатора зарядили от одинаковых источников. Первый конденсатор отключили от источника, а второй конденсатор оставили соединенным с источником. Пространство между обкладками обоих конденсаторов заполнили диэлектриком с диэлектрической проницаемостью, равной 3. Во сколько раз энергия второго конденсатора оказалась больше, чем у первого?

1) 3 2) 6 3) 8 4) 9 5) 12

Т5.180 Два конденсатора с емкостями 5 мкФ и 2 мкФ соединены последова-тельно и подключены к источнику напряжения. Найдите отношение энергии, запасенной в первом конденсаторе, к энергии во втором.

1) 2,5 2) 6,25 3) 0,2 4) 0,4 5) 0,16

 

Постоянный ток

 

©2015 arhivinfo.ru Все права принадлежат авторам размещенных материалов.

РАСЧЕТ НА

КВААР ДЛЯ УСИЛИТЕЛЯ — Искажение напряжения

Для определения максимальной токовой защиты часто необходимо рассчитать полный ток нагрузки конденсаторной батареи. При вычислении коэффициента мощности при полной нагрузке конденсатора интересно то, что необходимо учитывать множество параметров и переменных. Многие из этих параметров могут быть неизвестны в то время, что необходимо сделать инженерные оценки. Вот некоторые из переменных, определяющих ток конденсаторной батареи:

  • Допуск конденсатора
  • Допуск напряжения
  • Гармоники в системе

КВААР НА КАЛЬКУЛЯТОР AMPS — ТРЕХФАЗНЫЙ

КВААР К КАЛЬКУЛЯТОРУ УСИЛИТЕЛЯ — ОДНОФАЗНЫЙ

Например, ток конденсатора 25 кВАр можно рассчитать как 4 А для однофазной системы 7200 В с допуском конденсатора 10% и допуском напряжения 5%.

Калькулятор коэффициента мощности

Конденсатор продолжительный ток

Длительный основной ток однофазного конденсатора определяется по формуле:

Длительный основной ток трехфазного конденсатора определяется выражением:

Блок конденсаторов среднего напряжения 1200 кВАр. Каждый блок рассчитан на 400 кВАр при 7,2 кВ

Для системы, показанной на рисунке выше, конденсаторы рассчитаны на 400 кВАр при 7.2кВ. Отдельные конденсаторы соединены между нейтралью. Линейное напряжение системы составляет 12 470 В. Чистый рейтинг банка — 400 * 3 = 1,200кВАр. Чтобы рассчитать ток полной нагрузки, введите 1200 кВАр в качестве номинального значения, а напряжение — 12 470 В в трехфазном калькуляторе выше. При необходимости примените дополнительные допуски.

Другие факторы, влияющие на постоянный ток конденсатора

Несмотря на то, что ток конденсатора может быть рассчитан с использованием приведенных выше уравнений, он не будет очень точным из-за различных других факторов, влияющих на уравнение тока.Каждый из них обсуждается ниже:

Допуск конденсатора

IEEE STD 18-2012, который является стандартом для шунтирующих силовых конденсаторов, допускает допуск конденсатора в пределах 0-10%. Этот допуск может составлять + 15% в соответствии со стандартом IEC. Это означает, что конденсатор с данными на паспортной табличке 100 кВАр может обеспечивать реактивную мощность от 100 до 115 кВАр и, следовательно, потреблять больший ток.

Обычно можно получить производственный допуск от производителя или измерить емкость и определить допуск.

Допуск напряжения

Конденсаторы

предназначены для непрерывной работы при номинальном напряжении или ниже. Напряжение электросети редко бывает близко к номинальному значению, и отклонение +/- 5% считается нормальным. В некоторых местах и ​​на объектах может наблюдаться допуск даже +/- 10%.

Допуск напряжения установлен различными национальными стандартами, такими как ANSI C84.1 .

Конденсаторы

, соответствующие стандарту IEEE 18, могут работать при следующих условиях аварийного напряжения:

  • 110% от номинального действующего напряжения
  • 120% от номинального пикового напряжения

Выходная реактивная мощность конденсатора зависит от напряжения системы в соответствии со следующим уравнением:

Где Q 1 — реактивная мощность при напряжении V 1 , а Q 2 — реактивная мощность при напряжении V 2 .

Даже если конденсатор способен работать при перенапряжении 10%, он также потребляет соответствующий более высокий ток, который необходимо учитывать при расчете тока.

Допуск частоты

Изменение частоты влияет на поток реактивной мощности от конденсатора. Однако в современных электрических сетях изменение частоты незначительно и, следовательно, может быть проигнорировано при вычислении тока конденсатора.

Уравнение для расчета изменения реактивной мощности при изменении частоты питающей сети имеет следующий вид:

Где Q 1 — реактивная мощность с частотой f 1 , а Q 2 — реактивная мощность с частотой f 2 .

Гармоники

Когда конденсаторы помещаются в систему питания для коррекции коэффициента мощности, это изменяет поведение системы. Конденсатор представляет собой путь с низким сопротивлением для гармонических токов. Гармоническое напряжение, присутствующее в системе (из-за наличия нелинейных нагрузок), будет создавать дополнительный гармонический ток в конденсаторах. Этот ток может вызвать дополнительный нагрев и, в конечном итоге, привести к выходу конденсатора из строя, если он не спроектирован должным образом.

Чтобы учесть наличие неизбежных гармонических токов, допуск по напряжению и производственный допуск, в стандарте IEEE STD 18 указано, что конденсаторы должны работать при 135% номинального действующего значения тока на основе номинальной квар и номинального напряжения.

При расчете тока конденсатора рекомендуется включать номинал 135%, чтобы устройства защиты от перегрузки по току могли быть правильно подобраны.

Выбор кабелей конденсаторных батарей и устройств защиты от перегрузки по току

Как обсуждалось ранее, при выборе кабеля и устройства защиты от перегрузки по току для конденсаторных батарей необходимо учитывать следующие моменты:

  • Из-за допусков на изготовление конденсаторов емкость может варьироваться в пределах 0–10% [IEEE] или 0–15% [IEC] от значения, указанного на паспортной табличке.
  • Напряжение, при котором применяются конденсаторы, может изменяться от + 5% до + 10%. Напряжение ниже номинального не является проблемой, поскольку более низкое напряжение приведет к меньшему току конденсатора.
  • Гармоники могут создавать дополнительный ток в конденсаторах в диапазоне от + 20% до + 35% номинального тока.

Принимая во внимание все вышеперечисленные факторы, кабели и автоматический выключатель, предохранители должны быть рассчитаны.

В качестве примера, если мы рассмотрим 15% допуск конденсатора, 10% допуск по напряжению и 20% дополнительный ток из-за гармоник, тогда ток полной нагрузки конденсатора основной гармоники необходимо умножить на 1.15 * 1,10 * 1,20 = 1,518.

Типичные значения сечения кабелей и автоматических выключателей варьируются в 1,3–1,5 раза от номинального тока полной нагрузки конденсаторной батареи.

Дополнительное чтение:

Падение напряжения переменного тока и коэффициент мощности системы

Регулировка напряжения

Калькулятор коэффициента мощности

Как измерить реактивную мощность?

Как рассчитать реактивную мощность трансформатора?

Как рассчитать размер конденсаторной батареи в кварах?

Конденсаторы обычно делятся на две категории — коррекция коэффициента мощности и коррекция низкого напряжения.Аспект коррекции коэффициента мощности был рассмотрен выше. Влияние конденсаторов на уровень напряжения прямо пропорционально их номинальному значению в кВАр и индуктивному сопротивлению цепи. Это связано с помощью следующей формулы:

Повышение напряжения в процентах = (квар) (X) (l) ÷ ((10) (кВ) ²)

, где квар = 3Ø киловар
X = реактивное сопротивление в омах на единицу длины
l = длина цепи
кВ = Ø-Ø киловольт

В распределительной системе, где единица измерения длины выражается в милях, а конструкция основана на 8-футовых поперечинах, значение X на милю колеблется от 0.7 для проводов меньшего диаметра (№ 4, № 2) до 0,6 для более крупных (336,4). Таким образом, «быстрый» расчет для конденсатора 600 кВар на расстоянии 2 миль от источника в системе 11 кВ составляет приблизительно:

% VD ≈ 600 x 0,7 x 2 ÷ 1200 ≈ 0,7%

Аналогично, для системы 22 кВ приблизительная формула будет кВАр x 0,7 xl ÷ 5000. К сожалению, когда напряжение удваивается, влияние конденсатора уменьшается в 4 раза из-за кВ² в знаменателе. И значения в формуле можно легко преобразовать в метрические, взяв 5/8 импеданса (или приблизительно 0.4 Ом / километр).

Повышение напряжения в месте расположения банка будет применяться ко всей линии за пределами этой точки. Повышение напряжения между подстанцией и банком будет пропорционально расстоянию от подстанции.

Используя эту приблизительную формулу, легко произвести быстрое инженерное определение роста напряжения в результате их применения. Также обратите внимание, что для сельских распределительных линий более практичными являются банки размером от 150 до 600 кВАр. Банки мощностью от 1200 до 1800 кВА, расположенные далеко от подстанции, могут вызвать серьезный шок в системе при подаче напряжения.В сельских приложениях часто принято разделять банк и делать его наполовину фиксированным, а наполовину переключаемым. Конденсатор постоянной емкости может быть обесточен в непиковые месяцы, чтобы не допустить слишком высокого уровня напряжения.

Простые формулы конденсатора накопления энергии

У вас есть конденсатор, или вам нужно его выбрать, вы хотите вычислить некоторые вещи о нем с точки зрения его использования для хранения / доставки энергии (в отличие от фильтрации), вы хотели бы просто знать немного больше, чем онлайн-калькулятор, но не намного больше, потому что математика причиняет боль вашему мозгу.Эта страница для вас.

ln () (натуральный логарифм) часто встречается в уравнениях, натуральный логарифм — это обратное преобразование e в степень чего-либо (то есть ln (e x ) = x), в электронных таблицах это функция » ln () «, в коде (например, C / C ++ [Arduino!]), это обычно функция» log () «.
Все формулы предполагают «идеальный» конденсатор, без учета ESR или других неидеальных характеристик. Достаточно хорошо, чтобы попасть в бейсбольный стадион.
Вы можете изменить поля в каждом разделе, чтобы выполнить свой собственный расчет.

Помните, что ваше напряжение питания для зарядки конденсатора не должно превышать максимальное номинальное напряжение ваших конденсаторов (говоря в общих чертах).

У меня есть неизвестный конденсатор, известный резистор и секундомер, рассчитываю емкость.

C = (0 — секунды) / R / ln (1- (VCharged / VSupply))

Где секунды — это количество секунд, за которые взимается плата; R — резистор в Ом; VCharged — напряжение конденсатора в секундах; VSupply — это напряжение питания.

Вам не нужно заряжать конденсатор полностью, чтобы измерить его, если вы начинаете с разряда, рассчитываете период зарядки и записываете напряжение, которое вы достигли за этот период, вы можете выполнить расчет — но чем дольше (медленнее) вы заряжаете тем более точным будет ваш результат, потому что ваши ошибки и т. д. будут менее значимыми. Когда самая маленькая цифра на вашем счетчике, измеряющая напряжение конденсатора, изменяется один раз в секунду, это было бы разумным моментом для остановки.Имейте в виду также, что конденсаторы имеют заведомо большой допуск (+/- 30% вполне нормально для некоторых типов конденсаторов).

Вы можете использовать поля в примере для выполнения собственных расчетов, измените числа, чтобы увидеть, как себя ведут.

Сколько ампер-часов (Ач) в этом конденсаторе?

Ач = (C * (VCharged — VDepleted)) / 3600

Где VCharged — это напряжение заряда конденсатора, VDepleted — это опустошенное напряжение, а C — это емкость.

Здесь вы можете видеть, что если вы используете конденсатор для замены батареи, вам действительно нужно подключить его к преобразователю постоянного / постоянного тока с подходящим диапазоном входного напряжения, чтобы вы могли разрядить свой конденсатор до очень низкого напряжения, взяв наш В приведенном выше примере, если бы вместо напряжения отключения 3,3 В у нас было напряжение отключения 0,5 В, мы получили бы 10 мАч вместо жалких 2,5 мАч.

Вы можете использовать поля в примере для выполнения собственных расчетов, измените числа, чтобы увидеть, как себя ведут.

Пример

Конденсатор 10F, который был заряжен до 4,2В, разряжен до 3,3В, сколько там мАч?

(10 * (4,2 — 3,3)) / 3600 = 0,0025 Ач = 2,5 мАч

Сколько ватт-часов (Втч) в этом конденсаторе?

Вт · ч = (VCharged 2 — VDepleted 2 ) / (7200 / C)

Здесь вы можете видеть, что если вы используете конденсатор для замены батареи, вам действительно нужно подключить его к повышающему преобразователю с подходящим диапазоном входного напряжения, чтобы вы могли разрядить свой конденсатор до очень низкого напряжения, взяв наш пример выше. , если вместо 3.Напряжение отключения 3 В, у нас было напряжение отключения 0,5 В, мы получили бы 0,024 Вт-ч вместо мизерных 0,009 Вт-ч

Вы можете использовать поля в примере для выполнения собственных расчетов, измените числа, чтобы увидеть, как себя ведут.

Пример

Конденсатор 10F, который был заряжен до 4,2 В, разряжен до 3,3 В, сколько в нем Wh?

((4,2 2 ) — (3,3 2 )) / (7200/10) = 0,009375 Вт · ч

Сколько времени потребуется, чтобы зарядить этот конденсатор постоянным сопротивлением?

Секунды = 0 — (R * C * ln (1 — (VCharged / VSupply)))

Где VCharged — это напряжение, измеренное на конденсаторе, а VSupply — это напряжение источника питания, C — емкость в Фарадах, а R — резистор в Ом.

VCharged должно быть ниже VSupply — помните, что по мере того, как конденсатор заряжается больше, его сопротивление зарядке увеличивается, оно никогда не может достичь того же уровня, что и напряжение питания, даже если оно на неизмеримо меньше, оно всегда меньше.

Вы можете использовать поля в примере для выполнения собственных расчетов, измените числа, чтобы увидеть, как себя ведут.

Сколько времени потребуется, чтобы разрядить этот конденсатор через постоянное сопротивление?


Секунды = 0 — (R * C * ln (VDepleted / VCharged))

Где VCharged — начальное напряжение конденсатора, VDepleted — конечное напряжение, которое вы определите как пустое, R — сопротивление, C — емкость.

VDepleted должно быть больше нуля — помните, что ваша реальная схема, вероятно, не может много сделать с чем-либо, даже отдаленно близким к нулю.

Вы можете использовать поля в примере для выполнения собственных расчетов, измените числа, чтобы увидеть, как себя ведут.

Сколько времени потребуется, чтобы зарядить / разрядить этот конденсатор постоянным током?

Секунды = (C * (VCharged — VDepleted)) / Amps

Где C в фарадах, VCharged — это начальное напряжение на конденсаторе, VDepleted — это напряжение завершения разряда, а Amps — это ток в амперах.Для постоянного тока формула одинакова, независимо от того, разряжаете ли вы или заряжаете, разница в напряжении имеет значение, сколько напряжения должно нарастать или падать.

Вы можете использовать поля в примере для выполнения собственных расчетов, измените числа, чтобы увидеть, как себя ведут.

Пример

Конденсатор 10Ф разряжается с 5В до 4В при постоянном токе 500мА, сколько времени это занимает?

(10 * (5-4)) / 0,5 = 20 секунд (калькулятор)

Сколько времени потребуется, чтобы зарядить / разрядить этот конденсатор постоянной мощностью (Вт)?

Секунды = 0.5 * C * ((VCharged 2 — VDepleted 2 ) / Вт)

Где C — в фарадах, VS — это начальное напряжение на конденсаторе, VC — это напряжение завершения разряда, а P — мощность разряда в ваттах.

Вы можете использовать поля в примере для выполнения собственных расчетов, измените числа, чтобы увидеть, как себя ведут.

Пример

Конденсатор 10Ф разряжается с 5В до 4В при постоянной мощности 2Вт, сколько времени это занимает?

0.5 * 10 * ((5 2 -4 2 ) / 2) = 22,5 секунды

У меня есть аккумулятор / элемент на несколько ампер-часов. Сколько емкости мне нужно для непосредственной замены?

C = (Ач * 3600) / (VCharged — VDepleted)

Наивно мы можем предположить, что VCharged — это то же самое, что и номинальное напряжение вашей батареи, а VDepleted равно нулю, или, точнее говоря, VCharged — это максимальный заряд для вашей батареи, а VDepleted — это минимальное напряжение, которое ваша цепь может использовать.

Вы можете использовать поля в примере для выполнения собственных расчетов, измените числа, чтобы увидеть, как себя ведут.

Пример

Щелочной элемент емкостью 1250 мАч с полным напряжением 1,5 В и пустым напряжением 0,8 В должен быть заменен конденсатором, какого размера он должен быть?

(1,25 * 3600) / (1,5 — 0,8) = 6428 F

Очевидно, что это непрактично, поэтому см. Следующий раздел …

Если у меня есть батарея / элемент на несколько ампер-часов, какой емкости мне нужно заменить, если я использую преобразователь постоянного тока в постоянный?

C = 7200 / ((VCharged 2 — VDepleted 2 ) / ((Ah * VBattery) / 0.75))

Где Ah — это емкость батареи в Ач, VBattery — номинальное напряжение батареи, 0,75 — (наихудший случай) КПД преобразователя постоянного / постоянного тока, VCharged — это заряженное напряжение конденсатора, VDepleted — это наименьшее напряжение конденсатора вашего постоянного / постоянного тока. Преобразователь постоянного тока справится.

Вы можете использовать поля в примере для выполнения собственных расчетов, измените числа, чтобы увидеть, как себя ведут.

Пример

Щелочной элемент емкостью 1250 мАч с номинальным напряжением 1.5 В следует заменить конденсатором (батареей), который будет заряжаться до 10,8 В и приводится в действие понижающим преобразователем, который принимает входное напряжение до 1,6 В.

7200 / ((10,8 2 -1,6 2 ) / ((1,25 * 1,5) / 0,75)) = 157F

Я хочу рисовать x ампер в течение t секунд, какая емкость мне нужна?

C = (Амперы * секунды) / (VCharged — VDepleted)

Где C — требуемая емкость, Amps — это требуемый ток, VCharged — это начальное напряжение, до которого вы заряжали конденсатор, а VDepleted — это минимальное напряжение, которое вы будете принимать.Помните, как только вы потребляете ток из конденсатора, его напряжение падает, вот как это работает, поэтому вы не можете просто сказать: «Я хочу 1 ампер при X вольт», вы должны сказать, что я нарисую усилитель и может сделать это между этим и этим напряжением.

Вы можете использовать поля в примере для выполнения собственных расчетов, измените числа, чтобы увидеть, как себя ведут.

Пример

Вы хотите потреблять 500 мА от конденсатора, заряженного до 12 В, в течение 5 секунд, и после этого конденсатор будет измерять 9 В. Какого размера должен быть конденсатор?

(0.5 * 5) / (12 — 9) = 0,83F

Я хочу получать x Вт в течение t секунд, какая емкость мне нужна?

C = (секунды * 2) / ((VCharged 2 — VDepleted 2 ) / Watts)

Где C — емкость, Watts — мощность в ваттах, VCharged — это начальное напряжение, до которого вы заряжали конденсатор, а VDepleted — это минимальное напряжение, которое вы будете принимать. Помните, как только вы потребляете ток из конденсатора, его напряжение падает, вот как это работает, поэтому вы не можете просто сказать: «Я хочу 1 Вт при X Вольт», вы должны сказать, что я возьму ватт и может сделать это между этим и этим напряжением.

Вы можете использовать поля в примере для выполнения собственных расчетов, измените числа, чтобы увидеть, как себя ведут.

Пример

Вы хотите подавать 10 Вт в течение 5 секунд от конденсатора, первоначально заряженного до 12 В, а затем измеряя 9 В, какого размера должен быть конденсатор?

(5 * 2) / ((12 2 — 9 2 ) / 10) = 1,6F

Как вы пришли к этой формуле?

В представленной формуле нет ничего особенного. Хорошим справочником для упрощения работы является этот документ от ELNA, производителя суперконденсаторов, он охватывает основные уравнения для постоянного тока, мощности и разряда через сопротивление.

Electronics-Tutorials.ws обеспечивает разряд с постоянным сопротивлением, и заряд с постоянным сопротивлением также задается в виде Vc = Vs (1-e -t / RC ), которым можно управлять, чтобы найти t (см. Видео ниже) .

Это видео от Пола Уэсли Льюиса помогло моему лишенному математики мозгу научиться управлять манипуляциями.

Следующие ниже онлайн-калькуляторы были полезны при подтверждении моей работы Must Calculate, Circuits.dk, bitluni.net (ВНИМАНИЕ, расчет Wh на сайте bitluni неверен, если у вас минимальное напряжение> 0)

На основе этих уравнений и ресурсов получены следующие данные.

Вывод для ампер-часов

Начните с данной формулы для разряда при постоянном токе, установите t = 3600 секунд и решите, чтобы I было любым током, необходимым для разрядки конденсатора за это время и, следовательно, ампер-часов

секунд = (C * (VCharged — VDepleted)) / I

3600 = (C * (VCharged — VDepleted)) / I

I * 3600 = (C * (VCharged — VDepleted))

I = (C * (VCharged — VDepleted)) / 3600

(I = Ач)

Вывод для ватт-часов

Это выводится из формулы для разряда с постоянной мощностью, где t = 3600 секунд, вычисленных для P — любых ватт, необходимых для разряда конденсатора за это время и, следовательно, ватт-часов.

секунд = 0,5 * C * ((VCharged 2 — VDepleted 2 ) / P)

3600 = ((VCharged 2 — VDepleted 2 ) / P) * C * 0,5

3600 / 0,5 = ((VCharged 2 — VDepleted 2 ) / P) * C

7200 = ((VCharged 2 — VDepleted 2 ) / P) * C

7200 / C = (VCharged 2 — VDepleted 2 ) / P

P * (7200 / C) = (VCharged 2 — VDepleted 2 )

P = (VCharged 2 — VDepleted 2 ) / (7200 / C)

(P = Wh)

Вывод для эквивалентности батареи в ампер-часах

Это просто решение уравнения ампер-часов для емкости

Ач = (C * (VCharged — VDepleted)) / 3600

Ач * 3600 = C * (VCharged — VDepleted)

(Ач * 3600) / (VCharged — VDepleted) = C

Расчет эквивалентности батареи в ампер-часах с преобразователем постоянного тока в постоянный

Мы используем полученное выше уравнение ватт-часов, заменяя ватт-часы заданными ампер-часами и эквивалентным напряжением батареи, скорректированным с коэффициентом полезного действия 75% для повышающего преобразователя.

Вт · ч = (Vзаряжено 2 — VDepleted 2 ) / (7200 / C)

((Ач * VBattery) / 0,75) = (VCharged 2 — VDepleted 2 ) / (7200 / C)

7200 / C = (VCharged 2 — VDepleted 2 ) / (Ah * VBattery)

7200 = C * ((VCharged 2 — VDepleted 2 ) / (Ah * VBattery))

7200 / ((VCharged 2 — VDepleted 2 ) / (Ah * VBattery)) = C

Вывод для рисования ампер X для секунд T

Простое решение данного уравнения постоянного тока, решение для C

секунд = (C * (VCharged — VDepleted)) / I

секунд * I = C * (VCharged — VDepleted)

(секунды * I) / (VCharged — VDepleted) = C

Вывод для рисования X Вт для T секунд

Простое решение данного уравнения постоянной мощности, решение для C

секунд = 0.5 * C * ((VCharged 2 — VDepleted 2 ) / P)

Секунды = C * ((VCharged 2 — VDepleted 2 ) / P) * 0,5

Секунды * 2 = C * ((VCharged 2 — VDepleted 2 ) / P)

(секунды * 2) / ((VCharged 2 — VDepleted 2 ) / P) = C

::. IJSETR. ::

International Journal of Scientific Engineering and Technology Research (IJSETR) — международный журнал, предназначенный для профессионалов и исследователей во всех областях информатики и электроники.IJSETR публикует исследовательские статьи и обзоры по всей области инженерных наук и технологий, новые методы обучения, оценки, проверки и влияние новых технологий, и он будет продолжать предоставлять информацию о последних тенденциях и разработках в этой постоянно расширяющейся теме. Публикации статей отбираются путем двойного рецензирования, чтобы гарантировать оригинальность, актуальность и удобочитаемость. Статьи, опубликованные в нашем журнале, доступны онлайн.

Журнал объединит ведущих исследователей, инженеров и ученых в интересующей области со всего мира.Темы, представляющие интерес для представления, включают, помимо прочего:

• Электроника и связь
Машиностроение

• Электротехника

• Зеленая энергия и нанотехнологии

• Машиностроение

• Компьютерная инженерия

• Разработка программного обеспечения

• Гражданское строительство

• Строительное проектирование

• Строительное проектирование

• Электромеханическое машиностроение

• Телекоммуникационная техника

• Коммуникационная техника

• Химическая инженерия

• Пищевая промышленность

• Биологическая и биосистемная инженерия

• Сельскохозяйственная инженерия

• Инженерная геология

• Биомеханическая и биомедицинская инженерия

• Инженерные науки об окружающей среде

• Новые технологии и передовая инженерия

• Беспроводная связь и сетевое проектирование

• Тепловедение и инженерия

• Управление бизнесом, экономика и информационные технологии

• Органическая химия

• Науки о жизни, биотехнологии и фармацевтические исследования

• Тепло, Масстранфер и Технологии

• Биологические науки

• пищевая микробиология

• Сельскохозяйственные науки и технологии

• Водные ресурсы и экологическая инженерия

• Городские и региональные исследования

• Управление человеческими ресурсами

• Polution Engineering

• Математика

• Наука

• Астрономия

• Биохимия

• Биологические науки

• Химия

• Натуральные продукты

• Физика

• Зоология

• Наука о продуктах питания

• Материаловедение

• Прикладные науки

• Науки о Земле

• Универсальная аптека и LifeScience

• Квантовая химия

• Аптека

• Натуральные продукты и научные исследования

• Челюстно-лицевая и оральная хирургия

• Вопросы маркетинга и торговая политика

• Глобальный обзор деловых и экономических исследований

• управление бизнесом, экономика и информационные технологии

Особенность IJSETR…

• Прямая ссылка на аннотацию

• Открытый доступ для всех исследователей

• Автор может искать статью по названию, заголовку или ключевым словам

• Прямая ссылка на аннотацию к каждой статье

• Статистика по каждой статье как нет. раз его просмотрели и скачали

• Быстрый процесс публикации

• Предложение автору о доработке статьи

• Пост-публикация работает как индексация каждой статьи в разные базы данных.

• Журнал издается как в электронной, так и в печатной версии.

• Отправка печатной версии автору в течение недели после онлайн-версии

• Надлежащий процесс экспертной оценки

• Журнал предоставляет электронные сертификаты с цифровой подписью всем авторам после публикации статьи

• Полная статистика по каждому выпуску будет отображаться в ту же дату выпуска выпуска

энергии в конденсаторах | Физика

Цели обучения

К концу этого раздела вы сможете:

  • Перечислите некоторые варианты использования конденсаторов.
  • Выразите в уравнении энергию, запасенную в конденсаторе.
  • Объясните функцию дефибриллятора.

Большинство из нас видели инсценировки, в которых медицинский персонал использовал дефибриллятор , чтобы пропустить электрический ток через сердце пациента, чтобы заставить его нормально биться. (Просмотрите рис. 1.) Часто реалистичный в деталях, человек, применяющий электрошок, просит другого человека «сделать на этот раз 400 джоулей». Энергия, передаваемая дефибриллятором, накапливается в конденсаторе и может регулироваться в зависимости от ситуации.Часто используются единицы СИ — джоули. Менее драматично использование конденсаторов в микроэлектронике, например в некоторых портативных калькуляторах, для подачи энергии при зарядке аккумуляторов. (См. Рис. 1.) Конденсаторы также используются для питания ламп-вспышек на камерах.

Рис. 1. Энергия, накопленная в большом конденсаторе, используется для сохранения памяти электронного калькулятора, когда его батареи заряжены. (Источник: Kucharek, Wikimedia Commons)

Энергия, запасенная в конденсаторе, представляет собой электрическую потенциальную энергию, и, таким образом, она связана с зарядом Q и напряжением В на конденсаторе.Мы должны быть осторожны при применении уравнения для электрической потенциальной энергии ΔPE = q Δ V к конденсатору. Помните, что ΔPE — это потенциальная энергия заряда q , проходящего через напряжение Δ В . Но конденсатор начинает с нулевого напряжения и постепенно достигает своего полного напряжения по мере зарядки. Первый заряд, помещенный на конденсатор, испытывает изменение напряжения Δ В = 0, поскольку конденсатор имеет нулевое напряжение в незаряженном состоянии.Последний заряд, помещенный на конденсатор, испытывает Δ В = В , так как конденсатор теперь имеет на нем свое полное напряжение В . Среднее напряжение на конденсаторе во время процесса зарядки составляет [латекс] \ frac {V} {2} \\ [/ latex], поэтому среднее напряжение, испытываемое при полной зарядке q , составляет [латекс] \ frac {V} {2} \\ [/ латекс]. Таким образом, энергия, запасенная в конденсаторе E cap , равна [latex] E _ {\ text {cap}} = \ frac {QV} {2} \\ [/ latex], где Q — это заряд на конденсаторе с напряжением В приложено В.2} {2C} \\ [/ latex],

, где Q, — заряд, В, — напряжение, и C, — емкость конденсатора. Энергия выражается в джоулях для заряда в кулонах, напряжения в вольтах и ​​емкости в фарадах.

В дефибрилляторе доставка большого заряда коротким импульсом к набору лопастей на груди человека может быть спасением. Инфаркт у человека мог возникнуть в результате быстрого, нерегулярного сердцебиения — фибрилляции сердца или желудочков.Применение сильного разряда электрической энергии может прекратить аритмию и позволить кардиостимулятору тела вернуться к нормальному режиму. Сегодня в машинах скорой помощи обычно есть дефибриллятор, который также использует электрокардиограмму для анализа сердечного ритма пациента. Автоматические внешние дефибрилляторы (AED) можно найти во многих общественных местах (рис. 2). Они предназначены для использования непрофессионалами. Устройство автоматически диагностирует состояние сердца пациента, а затем применяет разряд с соответствующей энергией и формой волны.Во многих случаях перед использованием АВД рекомендуется СЛР.

Рис. 2. Автоматические внешние дефибрилляторы можно найти во многих общественных местах. Эти портативные устройства предоставляют устные инструкции по использованию в первые несколько важных минут для человека, страдающего сердечным приступом. (Источник: Оуайн Дэвис, Wikimedia Commons)

Пример 1. Емкость дефибриллятора сердца

Дефибриллятор сердца вырабатывает 4,00 × 10 2 Дж энергии за счет разряда конденсатора первоначально на 1.{-6} \ text {F} \\\ text {} & = & 8.00 \ mu \ text {F} \ end {array} \\ [/ latex]

Обсуждение

Это довольно большая, но управляемая емкость при 1,00 × 10 4 В.

Сводка раздела

  • Конденсаторы используются в различных устройствах, включая дефибрилляторы, микроэлектронику, такую ​​как калькуляторы, и лампы-вспышки, для подачи энергии. {2}} {2C} \\ [/ latex], где Q — это заряд, В, — напряжение, а C — емкость конденсатора.Энергия выражается в джоулях, когда заряд — в кулонах, напряжение — в вольтах, а емкость — в фарадах.

Концептуальные вопросы

  1. Как изменяется энергия, содержащаяся в заряженном конденсаторе, когда вставлен диэлектрик, если конденсатор изолирован и его заряд постоянен? Означает ли это, что работа была сделана?
  2. Что происходит с энергией, накопленной в конденсаторе, подключенном к батарее, когда вставлен диэлектрик? Была ли проделана работа в процессе?

Задачи и упражнения

  1. (a) Какая энергия хранится в 10.0 мкФ конденсатор дефибриллятора сердца заряжен до
    9,00 × 10 3 В? (b) Найдите количество сохраненного заряда.
  2. При операции на открытом сердце гораздо меньшее количество энергии вызывает дефибрилляцию сердца. (а) Какое напряжение приложено к конденсатору 8,00 мкФ дефибриллятора сердца, который накапливает 40,0 Дж энергии? (b) Найдите количество сохраненного заряда.
  3. Конденсатор емкостью 165 мкФ используется вместе с двигателем. Сколько энергии в нем хранится при подаче 119 В?
  4. Предположим, у вас есть 9.Батарея 00 В, конденсатор 2,00 мкФ и конденсатор 7,40 мкФ. (а) Найдите заряд и запасенную энергию, если конденсаторы подключены к батарее последовательно. (б) Сделайте то же самое для параллельного подключения.
  5. Нервный физик опасается, что две металлические полки его книжного шкафа с деревянным каркасом могут получить высокое напряжение, если они заряжены статическим электричеством, возможно, вызванным трением. (а) Какова емкость пустых полок, если они имеют площадь 1,00 × 10 2 м 2 и равны 0.200 м друг от друга? (б) Какое напряжение между ними, если на них помещены противоположные заряды величиной 2,00 нКл? (c) Чтобы показать, что это напряжение представляет небольшую опасность, рассчитайте запасенную энергию.
  6. Покажите, что для данного диэлектрического материала максимальная энергия, которую может хранить конденсатор с параллельными пластинами, прямо пропорциональна объему диэлектрика (Объем = A · d ). Обратите внимание, что приложенное напряжение ограничено диэлектрической прочностью.
  7. Создайте свою проблему. Рассмотрим дефибриллятор сердца, аналогичный описанному в примере 1. Постройте задачу, в которой вы исследуете заряд, накопленный в конденсаторе дефибриллятора, как функцию накопленной энергии. Среди факторов, которые необходимо учитывать, — это приложенное напряжение и то, должно ли оно меняться в зависимости от подаваемой энергии, диапазон задействованных энергий и емкость дефибриллятора. Вы также можете рассмотреть гораздо меньшую энергию, необходимую для дефибрилляции во время операции на открытом сердце, как вариант решения этой проблемы.
  8. Необоснованные результаты. (a) В определенный день для запуска двигателя грузовика требуется 9,60 × 10 3 Дж электроэнергии. Вычислите емкость конденсатора, способного хранить такое количество энергии при напряжении 12,0 В. (б) Что неразумного в этом результате? (c) Какие допущения ответственны?

Глоссарий

дефибриллятор: устройство, используемое для электрического разряда сердца пострадавшего от сердечного приступа с целью восстановления нормального ритмического паттерна сердца

Избранные решения проблем и упражнения

1.(а) 405 Дж; (б) 90,0 мС

2. (а) 3,16 кВ; (б) 25,3 мС

4. (а) 1.42 × 10 −5 C, 6.38 × 10 −5 Дж; (б) 8.46 × 10 −5 C, 3.81 × 10 −4 J

5. (а) 4,43 × 10 –12 F; б) 452 В; (в) 4.52 × 10 –7 Дж

8. (а) 133 F; (б) Такой конденсатор был бы слишком большим для перевозки в грузовике. Размер конденсатора был бы огромным; (c) Неразумно предполагать, что конденсатор может хранить необходимое количество энергии.

Емкостное реактивное сопротивление — обзор

X

Символ реактивного сопротивления.

X C

Символ емкостного реактивного сопротивления.

X L

Символ индуктивного реактивного сопротивления.

Демодуляция X и Z

Система демодуляции цветного ТВ, в которой два повторно вставлены 3.Сигналы поднесущей 58 МГц различаются примерно на 60 °, а не на обычные 90 °. Напряжения R-Y, B-Y и G-Y выводятся из демодулированных сигналов, и эти напряжения управляют тремя пушками кинескопа. Важным преимуществом этой системы является то, что схема приемника проще, чем требуется при I- и Q-демодуляции.

Ось X

1. Референтная ось в кристалле кварца. 2. Горизонтальная ось в системе прямоугольных координат.3. Направление по горизонтали или слева направо в двумерной системе координат. X-X означает одно направление, в котором следует метод пошагового повторения.

Диапазон X

Радиочастотный диапазон от 5200 до 11000 МГц с длинами волн от 5,77 до 2,75 см.

X-образный стержень

Прямоугольный кристаллический стержень, обычно вырезанный из Z-образного сечения, вытянутый параллельно X и с краями, параллельными X, Y и Z.

X-конденсатор

Конденсатор для подавления радиопомех, предназначенный для приложений, в которых отказ конденсатора не приведет к опасности поражения электрическим током.

Х-образный кристалл

Кристалл, вырезанный так, что его основные поверхности перпендикулярны электрической оси (X) исходного кристалла кварца.

ксенон

Инертный газ, используемый в некоторых тиратронах и других газовых трубках.

ксеноновая импульсная лампа

Источник некогерентного белого света высокой интенсивности; он работает, разряжая конденсатор через трубку с газом ксеноном. Такое устройство часто используется в качестве источника излучения накачки для различных лазеров с оптическим возбуждением.

ксерографический принтер

Устройство для печати оптического изображения на бумаге; светлые и темные области представлены электростатически заряженными и незаряженными участками на бумаге. Порошковые чернила, присыпанные пылью на бумаге, прилипают к заряженным участкам и впоследствии растворяются в бумаге под воздействием тепла.

ксерографическая запись

Ксерографическая запись.

ксерография

1.Эта ветвь электростатической электрофотографии, в которой изображения формируются на светопроводящей изолирующей среде с помощью инфракрасного, видимого или ультрафиолетового излучения. Затем среду присыпают порошком, который прилипает только к электростатически заряженному изображению. Затем применяется тепло, чтобы сплавить порошок в постоянное изображение. 2. Процесс печати электростатической электрофотографии, в котором используется светопроводящая изолирующая среда в сочетании с инфракрасным, видимым или ультрафиолетовым излучением для создания структур скрытого электростатического заряда для достижения наблюдаемой записи.

xeroprinting

Эта ветвь электростатической электрофотографии, в которой узор из изоляционного материала на проводящей среде используется для формирования структур электростатического заряда для использования при копировании.

xeroradiography

Процесс печати электростатической электрофотографии, в котором используется светопроводящая изолирующая среда в сочетании с рентгеновскими или гамма-лучами для создания структур скрытого электростатического заряда для достижения наблюдаемого рисунка.

ксерорадиографическое оборудование

Оборудование, использующее принципы электростатики и фотопроводимости для записи рентгеновских изображений на сенсибилизированную пластину через короткое время после экспонирования.

xfmr

Сокращенное обозначение трансформатора.

xistor

Сокращенное обозначение транзистора.

Разъем XLR

Экранированный трехжильный микрофонный штекер или розетка с фиксатором для разблокировки пальцами для предотвращения случайного извлечения.Стандартный разъем для профессиональных пользователей микрофонов.

xmitter

Аббревиатура передатчика. Также сокращенно trans или xmtr.

xmsn

Сокращенное обозначение передачи.

xmtr

Аббревиатура передатчика. Также сокращенно транс или xmitter.

X-off

Передатчик выключен.

X-on

Передатчик включен.

X-частица

Частица, имеющая такой же отрицательный заряд, что и электрон, но масса между электроном и протоном. Он создается космическим излучением, падающим на молекулы газа или фактически составляющим часть космических лучей.

Рентгеновский аппарат

Рентгеновская трубка и принадлежности к ней, включая рентгеновский аппарат.

Рентгеновская кристаллография

1. Использование рентгеновских лучей для изучения расположения атомов в кристалле.2. Изучение структуры кристаллических материалов с использованием взаимодействия рентгеновских лучей и электронной плотности кристалла (дифракции).

Устройство обнаружения рентгеновских лучей

Устройство, обнаруживающее неоднородности поверхности и объема твердых тел с помощью рентгеновских лучей.

Камера для дифракции рентгеновских лучей

Камера, которая направляет пучок рентгеновских лучей на образец неизвестного материала и позволяет полученным дифрагированным лучам воздействовать на полосу пленки.

Картина дифракции рентгеновских лучей

Картина, полученная на пленке, экспонированной с помощью рентгеновской дифракционной камеры. Он состоит из частей кругов с разным расстоянием между ними в зависимости от исследуемого материала.

Рентгеновский гониометр

Прибор, который определяет положение электрических осей кристалла кварца путем отражения рентгеновских лучей от атомных плоскостей кристалла.

Рентгеновские лучи

Также называемые рентгеновскими лучами.Проникающее излучение похоже на свет, но имеет гораздо более короткие длины волн (от 10 –7 до 10 –10 см). Обычно они возникают при бомбардировке металлической мишени потоком высокоскоростных электронов.

Рентгеновский спектрограф

Инструмент, который используется для построения диаграмм дифракции рентгеновских лучей, такой как рентгеновский спектрометр с фотографическими или другими регистрирующими устройствами.

Рентгеновский спектрометр

1.Прибор для получения рентгеновского спектра и измерения длин волн его компонентов. 2. Прибор, предназначенный для получения рентгеновского спектра материала в качестве помощи в его идентификации. Этот метод особенно полезен, когда материал не может быть физически разрушен.

Спектр рентгеновского излучения

Расположение пучка рентгеновских лучей в порядке длины волны.

Рентгеновский толщиномер

Бесконтактный толщиномер, используемый для измерения и индикации толщины движущегося холоднокатаного стального листа в процессе прокатки.Рентгеновский луч, направленный через лист, поглощается пропорционально толщине материала и его атомному номеру, и измерение количества поглощения дает непрерывное указание толщины листа.

Рентгеновская трубка

Вакуумная трубка, в которой рентгеновские лучи производятся путем бомбардировки мишени высокоскоростными электронами, ускоренными электростатическим полем.

Мишень для рентгеновской трубки

Также известен как антикатод.Электрод или электродная секция, на которую фокусируется электронный луч и который излучает рентгеновские лучи.

xso

Аббревиатура для кварцевого стабилизатора.

xtal

Аббревиатура кристалла.

X-волна

Одна из двух составляющих, на которые магнитное поле Земли делит радиоволну в ионосфере. Другой компонент — обыкновенная, или О-, волна.

Кристалл XY-огранки

Кристалл, ограненный таким образом, что его характеристики находятся между кристаллами X- и Y-огранки.

Плоттер XY

1. Устройство, используемое вместе с компьютером для нанесения координатных точек в виде графика. 2. Компьютерное устройство вывода, которое реагирует на цифровые сигналы предварительно записанных и / или обработанных данных путем распечатки линейных сегментов. Эти данные, которые могут включать буквенно-цифровые символы, диаграммы, таблицы или рисунки, загружаются из памяти компьютера со скоростью, достаточной для работы плоттера.XY-плоттер нельзя использовать для прямой записи аналоговых сигналов без дигитайзеров.

Регистратор XY

1. Регистратор, который отслеживает на графике взаимосвязь между двумя переменными, ни одна из которых не является временем. Иногда диаграмма перемещается, и одна из переменных контролируется так, что взаимосвязь действительно увеличивается пропорционально времени. 2. Регистратор, в котором два сигнала одновременно записываются одним пером, которое приводится в движение в одном направлении (ось X) одним сигналом, а в другом направлении (ось Y) — вторым сигналом.3. Регистратор данных, который используется для записи изменения одного параметра по отношению к другому. Например, изменение давления в зависимости от температуры. Для этих самописцев доступен широкий спектр преобразователей для преобразования физических параметров в электрические сигналы, используемые в самописце. Датчики давления, термопары, тензодатчики и акселерометры — вот несколько примеров. 4. Тип регистратора, который реагирует на поступающие аналоговые сигналы по мере их появления. Сигналы печатаются на графике заранее определенного размера, который может охватывать тестовые периоды от нескольких секунд до целого года.XY-рекордер записывает непрерывные линии. Кроме того, скорость отклика прибора важна для точности записи.

Переключатель XY

Переключатель с дистанционным управлением, расположенный так, что дворники перемещаются вперед и назад по горизонтали.

Знакомство с конденсаторными схемами коррекции коэффициента мощности — Блог о пассивных компонентах

Источник: блог Capacitor Faks

Часть мощности переменного тока, потребляемой индуктивными нагрузками, используется для поддержания инверсии магнитного поля из-за фазового сдвига между током и напряжением.Эту энергию можно рассматривать как потерянную энергию, поскольку она не используется для выполнения полезной работы. Цепи коррекции коэффициента мощности используются для минимизации реактивной мощности и повышения эффективности, с которой индуктивные нагрузки потребляют мощность переменного тока. Конденсаторы являются важными компонентами в схемах компенсации коэффициента мощности, и в этой статье будут рассмотрены некоторые конструктивные особенности при использовании этих компонентов для коррекции коэффициента мощности.

Реактивная мощность в индуктивных нагрузках

Индуктивные нагрузки, такие как дроссели, двигатели, оборудование для индукционного нагрева, генераторы, трансформаторы и оборудование для дуговой сварки, создают электрическую задержку, которую обычно называют индуктивностью.Эта индуктивность вызывает разность фаз между током и напряжением. На рис. 1 показаны формы сигналов тока и напряжения для нагрузки с нулевым запаздыванием (чисто резистивная нагрузка).

Рисунок 1 Напряжение и ток для идеальной нагрузки

Из-за фазового сдвига из-за индуктивности бывают моменты, когда ток и напряжение имеют разные знаки. В это время генерируется отрицательная энергия, которая возвращается в сеть электроснабжения. Когда два возвращают одинаковый знак, для генерации магнитных полей требуется аналогичное количество энергии.Энергия, которая теряется из-за перемагничивания в индуктивных нагрузках, обычно называется реактивной мощностью.

Индуктивные нагрузки переменного тока подразделяются на линейные и нелинейные устройства. Для линейных нагрузок форма сигнала тока и форма сигнала напряжения имеют совпадающие синусоидальные профили. На рисунке 2 показаны кривые тока и напряжения для типичной линейной нагрузки. С другой стороны, поскольку нелинейные нагрузки потребляют ток на разных частотах, формы сигналов тока и напряжения различаются.Для большинства нелинейных нагрузок форма сигнала тока обычно несинусоидальная. На рис. 3 показаны кривые тока и напряжения для нелинейной нагрузки.

Рисунок 2 Напряжение и ток для линейной нагрузки

Рисунок 3 Напряжение и ток для нелинейной нагрузки

Некоторые примеры линейных электрических нагрузок включают нагревательное оборудование, двигатели и лампы накаливания. К нелинейным устройствам относятся частотно-регулируемые приводы, приводы постоянного тока, программируемые контроллеры, осветительные устройства дугового типа, индукционные печи, источники бесперебойного питания и персональные компьютеры.Известно, что нелинейные электрические нагрузки являются основной причиной гармонических искажений в системах распределения электроэнергии.

Коэффициент мощности

Эффективность, с которой электрические устройства или установки потребляют мощность переменного тока, варьируется. Некоторые нагрузки используют мощность эффективно, в то время как другие тратят значительную часть потребляемой мощности. Коэффициент мощности используется для описания эффективности, с которой нагрузки потребляют мощность переменного тока. Эта безразмерная величина находится в диапазоне от 0 до 1.

Как показано на рис. 4 и рис. 5 , общая мощность переменного тока, также известная как полная мощность, потребляемая электрическим устройством или оборудованием, зависит от двух компонентов: полезной мощности (активной мощности) и реактивной мощности. Под полезной мощностью понимается мощность, необходимая устройству для выполнения задачи. С другой стороны, реактивная мощность не дает полезной работы. Полезная мощность обычно измеряется в кВт, а реактивная мощность — в кВАр.

Рисунок 4 и 5, активная и реактивная мощности диаграммы полной полной мощности

Как показано в уравнении 1 , коэффициент мощности равен отношению активной мощности (полезной мощности) к общей мощности (полной мощности), потребляемой электрическим устройством или оборудованием.Математически можно показать, что коэффициент мощности равен косинусу угла θ ( Уравнение 2 ). Чем ближе это отношение к 1,0, тем выше эффективность устройства или оборудования.

Для идеальной электрической нагрузки коэффициент мощности равен 1,0 (единичный коэффициент мощности). Это означает, что вся мощность, потребляемая нагрузкой, используется для выполнения полезной работы. Однако реальной электрической нагрузке добиться этого сложно. Импеданс для нагрузки, представленной , рис. 5, определяется уравнением 3, где XL — индуктивное реактивное сопротивление, которое определяется уравнением , уравнением 4 .

Почему электрической нагрузке трудно достичь единичного коэффициента мощности? Большинству электрических нагрузок присущи реактивные свойства, которые затрудняют достижение идеального коэффициента мощности. Чтобы преодолеть это ограничение, в сеть добавляются схемы коррекции коэффициента мощности для компенсации реактивных характеристик нагрузки.

Коррекция коэффициента мощности (компенсация)

Электрические нагрузки с низким коэффициентом мощности потребляют больше энергии, чем необходимо для выполнения задачи.Это может привести к значительным потерям мощности в сети и высоким потерям в трансформаторе. Такое увеличение потребления энергии увеличивает стоимость работающего оборудования или установок. Низкие коэффициенты мощности также вызывают повышенные падения напряжения в распределительной сети. Поставщики электроэнергии обычно наказывают отрасли, коэффициент мощности которых ниже установленного значения.

Поставщики электроэнергии побуждают промышленных потребителей повышать коэффициент мощности по разным причинам. Во-первых, повышение коэффициента мощности может помочь значительно сократить расходы на электроэнергию.Во-вторых, высокий коэффициент мощности помогает минимизировать потери КПД в трансформаторах потребителя. В-третьих, добавление системы коррекции коэффициента мощности помогает увеличить эффективную мощность электрической сети потребителя. Наконец, высокий коэффициент мощности способствует увеличению срока службы электрооборудования.

Сеть компенсации коэффициента мощности снижает мощность, потребляемую нагрузкой, тем самым улучшая общий коэффициент мощности. Компенсационная сеть позволяет электрическим нагрузкам достигать хорошего коэффициента мощности, обычно от 0.95 и 0,98. Коэффициент мощности 0,85 и ниже обычно рассматривается коммунальными предприятиями как плохой коэффициент мощности.

Цепи конденсаторной коррекции коэффициента мощности

Существуют различные методы повышения коэффициента мощности нагрузки или установки. Один из часто используемых методов включает добавление в сеть конденсаторов для коррекции коэффициента мощности. На рисунке 6 показана простая схема, состоящая из источника переменного тока и индуктивной нагрузки.

Рисунок 6 и 7, индуктивная нагрузка с конденсатором коррекции коэффициента мощности и без него

Как конденсатор помогает улучшить коэффициент мощности? В цепи переменного тока реверсирование магнитного поля из-за разности фаз между током и напряжением происходит 50 или 60 раз в секунду.Конденсатор помогает улучшить коэффициент мощности, освобождая линию питания от реактивной мощности. Конденсатор достигает этого за счет накопления энергии обратного магнитного поля.

На рисунке 7 показана индуктивная нагрузка с конденсатором коррекции коэффициента мощности. На рисунке 8 выше показано улучшение коэффициента мощности при добавлении конденсатора в схему. Импеданс для цепи с конденсатором компенсации коэффициента мощности определяется уравнением , уравнение 5, , где XC — емкостное реактивное сопротивление, которое определяется уравнением , уравнением 6, .

В большинстве отраслей для компенсации реактивной мощности устанавливается система конденсаторов, управляемая контроллером коррекции коэффициента мощности. При проектировании системы коррекции коэффициента мощности важно избегать увеличения емкости сети. Добавление избыточной емкости к цепи может привести к чрезмерной коррекции, как показано на Рис. 9.

Полупроводниковые приборы также широко используются для коррекции коэффициента мощности. Использование полупроводниковых устройств в цепи для улучшения коэффициента мощности обычно называется активной компенсацией.Синхронные машины с перевозбуждением также обычно используются для улучшения коэффициента мощности сети.

Заключение

Индуктивные нагрузки, такие как трансформаторы, генераторы, двигатели, дроссели и оборудование для дуговой сварки, создают электрическую задержку, в результате чего ток и напряжение имеют разные знаки. Энергия, необходимая для поддержания разворота магнитного поля в индуктивных нагрузках, называется реактивной мощностью.

Разное

Добавить комментарий

Ваш адрес email не будет опубликован. Обязательные поля помечены *